Unit 5 TB

अब Quizwiz के साथ अपने होमवर्क और परीक्षाओं को एस करें!

7. A nurse in the CCU is caring for a patient with HF who has developed an intracardiac thrombus. This creates a high risk for what sequela? A) Stroke B) Myocardial infarction (MI) C) Hemorrhage D) Peripheral edema - 567

: A Feedback: Intracardiac thrombi can become lodged in the cerebral vasculature, causing stroke. There is no direct risk of MI, hemorrhage, or peripheral edema.

36. The nurse is caring for a patient who has developed obvious signs of pulmonary edema. What is the priority nursing action? A) Lay the patient flat. B) Notify the family of the patients critical state. C) Stay with the patient. D) Update the physician. - 580

: C Feedback: Because the patient has an unstable condition, the nurse must remain with the patient. The physician must be updated promptly, but the patient should not be left alone in order for this to happen. Supine positioning is unlikely to relieve dyspnea. The family should be informed, but this is not the priority action.

A nurse is caring for a patient who is exhibiting ventricular tachycardia (VT). Because the patient is pulseless, the nurse should prepare for what intervention? A)Defibrillation B)ECG monitoring C)Implantation of a cardioverter defibrillator D)Angioplasty

A

A patient is undergoing preoperative teaching before his cardiac surgery and the nurse is aware that a temporary pacemaker will be placed later that day. What is the nurse's responsibility in the care of the patient's pacemaker? A)Monitoring for pacemaker malfunction or battery failure B)Determining when it is appropriate to remove the pacemaker C)Making necessary changes to the pacemaker settings D)Selecting alternatives to future pacemaker use

A

A patient the nurse is caring for has a permanent pacemaker implanted with the identification code beginning with VVI. What does this indicate? A)Ventricular paced, ventricular sensed, inhibited B)Variable paced, ventricular sensed, inhibited C)Ventricular sensed, ventricular situated, implanted D)Variable sensed, variable paced, inhibited

A

A patient who is a candidate for an implantable cardioverter defibrillator (ICD) asks the nurse about the purpose of this device. What would be the nurse's best response? A)"To detect and treat dysrhythmias such as ventricular fibrillation and ventricular tachycardia" B)"To detect and treat bradycardia, which is an excessively slow heart rate" C)"To detect and treat atrial fibrillation, in which your heart beats too quickly and inefficiently" D)"To shock your heart if you have a heart attack at home"

A

A patient with advanced venous insufficiency is confined following orthopedic surgery. How can the nurse best prevent skin breakdown in the patient's lower extremities? A) Ensure that the patient's heels are protected and supported. B) Closely monitor the patient's serum albumin and prealbumin levels. C) Perform gentle massage of the patient's lower legs, as tolerated. D) Perform passive range-of-motion exercises once per shift.

A

The nurse and the other members of the team are caring for a patient who converted to ventricular fibrillation (VF). The patient was defibrillated unsuccessfully and the patient remains in VF. According to national standards, the nurse should anticipate the administration of what medication? A)Epinephrine 1 mg IV push B)Lidocaine 100 mg IV push C)Amiodarone 300 mg IV push D)Sodium bicarbonate 1 amp IV push

A

The nurse is caring for a patient who has had a biventricular pacemaker implanted. When planning the patient's care, the nurse should recognize what goal of this intervention? A)Resynchronization B)Defibrillation C)Angioplasty D)Ablation

A

The nurse is caring for a patient who has had an ECG. The nurse notes that leads I, II, and III differ from one another on the cardiac rhythm strip. How should the nurse best respond? A)Recognize that the view of the electrical current changes in relation to the lead placement. B)Recognize that the electrophysiological conduction of the heart differs with lead placement. C)Inform the technician that the ECG equipment has malfunctioned. D)Inform the physician that the patient is experiencing a new onset of dysrhythmia.

A

The nurse is caring for a patient who has just undergone catheter ablation therapy. The nurse in the step-down unit should prioritize what assessment? A)Cardiac monitoring B)Monitoring the implanted device signal C)Pain assessment D)Monitoring the patient's level of consciousness (LOC)

A

The nurse is caring for a patient with a large venous leg ulcer. What intervention should the nurse implement to promote healing and prevent infection? A) Provide a high-calorie, high-protein diet. B) Apply a clean occlusive dressing once daily and whenever soiled. C) Irrigate the wound with hydrogen peroxide once daily. D) Apply an antibiotic ointment on the surrounding skin with each dressing change.

A

advanced age

A community health nurse is giving an educational presentation about stroke and heart disease at the local senior citizens center. What nonmodifiable risk factor for stroke should the nurse cite?

stop smoking as soon as possible

A family member brings the patient to the clinic for a follow up visit after a stroke. The family member asks the nurse what he can do to decrease his chance of having another stroke. What would be the nurses best answer?

place the patients extremities where she can see them

A female pt is dx with a right side stroke. The patient is now experiencing hemianopsia. How might the nurse help the patient manage her potential sensory & perceptional difficulties?

report this to the physician as a possible sign of clinical deterioration

A nurse in the ICU is providing care for a patient who has been admitted with a hemorrhagic stroke. The nurse is performing frequent neurologic assessments and observes that the patient is becoming progressively more drowsy over the course of the day. what is the nurses best response to this assessment finding?

maintain and improve cerebral tissue perfusion

A nurse is caring for a patient dx with hemorrhagic stroke. When creating this patients plan of care, what goals should be prioritized?

disturbed sensory perception

A nursing student is writing a care plan for a newly admitted patient who has been diagnosed with a stroke. Whatt major nursing diagnosis should most likely be included in the patients plan of care?

to remove atherosclerotic plaques blocking cerebral flow

A patient diagnosed with transient ischemic attacks (TIAs) is scheduled for a carotid endarterectomy. The nurse explains that this procedure will be done for what purpose?

call the physician immediately

A patient dx with a cerebral aneurysm reports a severe headache to the nurse. What action is a priority for the nurse?

the pt should be taught to interlace fingers, place palms together, and slowly bring scapulae forward to avoid excessive force to shoulder

A patient recovering from a stroke has severe shoulder pain from subluxation of the shoulder and is being cared for on the unit. To prevent further injury and pain, the nurse caring for this pt is aware of what principle of care?

cardiac & respiratory status

A patient who just suffered a suspected ischemic stroke is brought to the ED by ambulance. on what should the nurses primary assessment focus?

absolute bed rest in a quiet, nonstimulating environment

A patient with a cerebral aneurysm exhibits signs and symptoms of an increase in ICP. what nursing intervention would be most appropriate for this pt?

A-fib

A preceptor is discussing stroke with a new nurse on the unit. The preceptor would tell the new nurse which cardiac dysrhythmia is associated with cardiogenic embolic strokes?

evidence of hemorrhagic stroke

A pt is brought by ambulance to the ED after suffering what the family thinks is a stroke. The nurse caring for this pt is aware that an absolute contraindication for thrombolytic therapy is what?

The nurse is evaluating a patient's diagnosis of arterial insufficiency with reference to the adequacy of the patient's blood flow. On what physiological variables does adequate blood flow depend? Select all that apply. A) Efficiency of heart as a pump B) Adequacy of circulating blood volume C) Ratio of platelets to red blood cells D) Size of red blood cells E) Patency and responsiveness of the blood vessels

A, B, E

The nurse has performed a thorough nursing assessment of the care of a patient with chronic leg ulcers. The nurses assessment should include which of the following components? Select all that apply. A) Location and type of pain B) Apical heart rate C) Bilateral comparison of peripheral pulses D) Comparison of temperature in the patients legs E) Identification of mobility limitations

A,C,D, E

27. During an adult patient's last two office visits, the nurse obtained BP readings of 122/84 mm Hg and 130/88 mm Hg, respectively. How would this patient's BP be categorized? A) Normal B) Prehypertensive C) Stage 1 hypertensive D) Stage 2 hypertensive

Ans: B Feedback: Prehypertension is defined systolic BP of 120 to 139 mm Hg or diastolic BP of 80 to 89 mm Hg.

11. The nurse is planning the care of a patient who has been diagnosed with hypertension, but who otherwise enjoys good health. When assessing the response to an antihypertensive drug regimen, what blood pressure would be the goal of treatment? A) 156/96 mm Hg or lower B) 140/90 mm Hg or lower C) Average of 2 BP readings of 150/80 mm Hg D) 120/80 mm Hg or lower

Ans: B Feedback: The goal of antihypertensive drug therapy is a BP of 140/90 mm Hg or lower. A pressure of 130/80 mm Hg is the goal for patients with diabetes or chronic kidney disease.

39. A patient's medication regimen for the treatment of hypertension includes hydrochlorothiazide. Following administration of this medication, the nurse should anticipate what effect? A) Drowsiness or lethargy B) Increased urine output C) Decreased heart rate D) Mild agitation

Ans: B Feedback: Thiazide diuretics lower BP by reducing circulating blood volume; this results in a short-term increase in urine output. These drugs do not cause bradycardia, agitation, or drowsiness.

19. The nurse is providing care for a patient with a diagnosis of hypertension. The nurse should consequently assess the patient for signs and symptoms of which other health problem? A) Migraines B) Atrial-septal defect C) Atherosclerosis D) Thrombocytopenia

Ans: C Feedback: Hypertension is both a sign and a risk factor for atherosclerotic heart disease. It is not associated with structural cardiac defects, low platelet levels, or migraines.

38. A patient in hypertensive urgency is admitted to the hospital. The nurse should be aware of what goal of treatment for a patient in hypertensive urgency? A) Normalizing BP within 2 hours B) Obtaining a BP of less than 110/70 mm Hg within 36 hours C) Obtaining a BP of less than 120/80 mm Hg within 36 hours D) Normalizing BP within 24 to 48 hours

Ans: D Feedback: In cases of hypertensive urgency, oral agents can be administered with the goal of normalizing BP within 24 to 48 hours. For patients with this health problem, a BP of 120/80 mm Hg may be unrealistic.

The ED nurse is caring for a patient with a suspected MI. What drug should the nurse anticipate administering to this patient? A) Oxycodone B) Warfarin C) Morphine D) Acetaminophen

Ans: Morphine Feedback: The patient with suspected MI is given aspirin, nitroglycerin, morphine, an IV beta- blocker, and other medications, as indicated, while the diagnosis is being confirmed. Tylenol, warfarin, and oxycodone are not typically used.

A nurse has written a plan of care for a man diagnosed with peripheral arterial insufficiency. One of the nursing diagnoses in the care plan is altered peripheral tissue perfusion related to compromised circulation. What is the most appropriate intervention for this diagnosis? A) Elevate his legs and arms above his heart when resting. B) Encourage the patient to engage in a moderate amount of exercise. C) Encourage extended periods of sitting or standing. D) Discourage walking in order to limit pain.

B

The nurse is analyzing a rhythm strip. What component of the ECG corresponds to the resting state of the patient's heart? A)P wave B)T wave C)U wave D)QRS complex

B

A nurse on a medical unit is caring for a patient who has been diagnosed with lymphangitis. When reviewing this patient's medication administration record, the nurse should anticipate which of the following? A) Coumadin (warfarin) B) Lasix (furosemide) C) An antibiotic D) An antiplatelet aggregator

C

The nurse is planning discharge teaching for a patient with a newly inserted permanent pacemaker. What is the priority teaching point for this patient? A)Start lifting the arm above the shoulder right away to prevent chest wall adhesion. BAvoid cooking with a microwave oven. C)Avoid exposure to high-voltage electrical generators. D)Avoid walking through store and library antitheft devices.

C

The nurse is preparing to administer warfarin (Coumadin) to a client with deep vein thrombophlebitis (DVT). Which laboratory value would most clearly indicate that the patient's warfarin is at therapeutic levels? A) Partial thromboplastin time (PTT) within normal reference range B) Prothrombin time (PT) eight to ten times the control C) International normalized ratio (INR) between 2 and 3 D) Hematocrit of 32%

C

A cardiac care nurse is aware of factors that result in positive chronotropy. These factors would affect a patient's cardiac function in what way? A)Exacerbating an existing dysrhythmia B)Initiating a new dysrhythmia C)Resolving ventricular tachycardia D)Increasing the heart rate

D

-national institutes of health stroke scale (NIHSS) -LOC at time of admission -age

During a patients recovery from stroke, the nurse should be aware of predictors of stroke outcome in order to help patients and families set realistic goals. What are the predictors of stroke outcome? *select all that apply*

facial droop

The nurse is assessing a patient with a suspected stroke. What assessment finding is most suggestive of a stroke?

the pt should be placed in a prone position for 15 to 30 min several times a day

The nurse is caring for a patient diagnosed with an ischemic stroke and knows that effective positioning of the patient is important. Which of the following should be integrated into the patient plan of care?

elevation of the head of the bed

When caring for a patient who has had a stroke, a priority is reduction of ICP. What pt position is most consistent with this goal?

how to correctly modify the home environment

a patient diagnosed with a hemorrhagic stroke has been admitted to the neuro ICU. the nurse knows that teaching for the patient and family needs to begin as soon as the patient is settled on the unit and will continue until the patient is discharged. What will family education need to include?

teaching the patient to perform deep breathing & coughing exercises

the nurse is caring for a patient recovering from an ischemic stroke. What intervention best addresses a potential complication after an ischemic stroke?

depression

the nurse is discharging home a patient who suffered a stroke. He has a flaccid right arm and leg and is experiencing problems with urinary incontinence. The nurse makes a referral to a home health nurse because of an awareness of what common patient response to a change in body image?

exercise the affected extremities passively four or five times a day

what should be included in the patients care plan when establishing an exercise program for a patient affected by a stroke?

ischemic pathway

-blood flow decreases -a switch to anaerobic respiration -lactic acid is generated -change pH -membrane pumps fail -cells cease to function

38. A patient who is at high risk for developing intracardiac thrombi has been placed on long-term anticoagulation. What aspect of the patients health history creates a heightened risk of intracardiac thrombi? A) Atrial fibrillation B) Infective endocarditis C) Recurrent pneumonia D) Recent surgery

: A Feedback: - 581 Intracardiac thrombi are especially common in patients with atrial fibrillation, because the atria do not contract forcefully and blood flows slowly and turbulently, increasing the likelihood of thrombus formation. Endocarditis, pneumonia, and recent surgery do not normally cause an increased risk for intracardiac thrombi formation.

6. The nurse is caring for an adult patient with HF who is prescribed digoxin. When assessing the patient for adverse effects, the nurse should assess for which of the following signs and symptoms? A) Confusion and bradycardia B) Uncontrolled diuresis and tachycardia C) Numbness and tingling in the extremities D) Chest pain and shortness of breath

: A Feedback: A key concern associated with digitalis therapy is digitalis toxicity. Symptoms include anorexia, nausea, visual disturbances, confusion, and bradycardia. The other listed signs and symptoms are not characteristic of digitalis toxicity.

25. A patient with a diagnosis of HF is started on a beta-blocker. What is the nurses priority role during gradual increases in the patients dose? A) Educating the patient that symptom relief may not occur for several weeks B) Stressing that symptom relief may take up to 4 months to occur C) Making adjustments to each days dose based on the blood pressure trends D) Educating the patient about the potential changes in LOC that may result from the drug - 575

: A Feedback: An important nursing role during titration is educating the patient about the potential worsening of symptoms during the early phase of treatment and stressing that improvement may take several weeks. Relief does not take 4 months, however. The nurse monitors blood pressure, but changes are not made based on short-term assessment results. Beta-blockers rarely affect LOC.

28. The nurse is performing an initial assessment of a client diagnosed with HF. The nurse also assesses the patients sensorium and LOC. Why is the assessment of the patients sensorium and LOC important in patients with HF? A) HF ultimately affects oxygen transportation to the brain. B) Patients with HF are susceptible to overstimulation of the sympathetic nervous system. C) Decreased LOC causes an exacerbation of the signs and symptoms of HF. D) The most significant adverse effect of medications used for HF treatment is altered LOC.

: A Feedback: As the volume of blood ejected by the heart decreases, so does the amount of oxygen transported to the brain. Sympathetic stimulation is not a primary concern in patients with HF, although it is a possibility. HF affects LOC but the reverse is not usually true. Medications used to treat HF carry many adverse effects, but the most common and significant effects are cardiovascular.

29. Cardiopulmonary resuscitation has been initiated on a patient who was found unresponsive. When performing chest compressions, the nurse should do which of the following? A) Perform at least 100 chest compressions per minute. B) Pause to allow a colleague to provide a breath every 10 compressions. C) Pause chest compressions to allow for vital signs monitoring every 4 to 5 minutes. D) Perform high-quality chest compressions as rapidly as possible.

: A Feedback: During CPR, the chest is compressed 2 inches at a rate of at least 100 compressions per minute. This rate is the resuscitators goal; the aim is not to give compressions as rapidly as possible. Compressions are not stopped after 10 compressions to allow for a breath or for full vital signs monitoring. - 577

40. The nurse is caring for a patient with severe left ventricular dysfunction who has been identified as being at risk for sudden cardiac death. What medical intervention can be performed that may extend the survival of the patient? A) Insertion of an implantable cardioverter defibrillator B) Insertion of an implantable pacemaker C) Administration of a calcium channel blocker D) Administration of a beta-blocker

: A Feedback: In patients with severe left ventricular dysfunction and the possibility of life-threatening dysrhythmias, placement of an implantable cardioverter defibrillator (ICD) can prevent sudden cardiac death and extend survival. A pacemaker, a calcium channel blocker, and a beta-blocker are not medical interventions that may extend the survival of the patient with left ventricular dysfunction.

35. A cardiovascular patient with a previous history of pulmonary embolism (PE) is experiencing a sudden onset of dyspnea, rapid breathing, and chest pain. The nurse recognizes the characteristic signs and symptoms of a PE. What is the nurses best action? A) Rapidly assess the patients cardiopulmonary status. B) Arrange for an ECG. C) Increase the height of the patients bed. D) Manage the patients anxiety.

: A Feedback: Patient management in the event of a PE begins with cardiopulmonary assessment and intervention. This is a priority over ECG monitoring, management of anxiety, or repositioning of the patient, even though each of these actions may be appropriate and necessary.

22. A patient with HF has met with his primary care provider and begun treatment with an angiotensin- converting enzyme (ACE) inhibitor. When the patient begins treatment, the nurse should prioritize what assessment? A) Blood pressure B) Level of consciousness (LOC) C) Assessment for nausea D) Oxygen saturation

: A Feedback: Patients receiving ACE inhibitors are monitored for hypotension, hyperkalemia (increased potassium in the blood), and alterations in renal function. ACE inhibitors do not typically cause alterations in LOC. Oxygen saturation must be monitored in patients with HF, but this is not particular to ACE inhibitor therapy. ACE inhibitors do not normally cause nausea.

30. The nurse is providing patient education prior to a patients discharge home after treatment for HF. The nurse gives the patient a home care checklist as part of the discharge teaching. What should be included on this checklist? A) Know how to recognize and prevent orthostatic hypotension. B) Weigh yourself weekly at a consistent time of day. C) Measure everything you eat and drink until otherwise instructed. D) Limit physical activity to only those tasks that are absolutely necessary.

: A Feedback: Patients with HF should be aware of the risks of orthostatic hypotension. Weight should be measured daily; detailed documentation of all forms of intake is not usually required. Activity should be gradually increased within the parameters of safety and comfort.

11. A patient admitted to the medical unit with HF is exhibiting signs and symptoms of pulmonary edema. The nurse is aware that positioning will promote circulation. How should the nurse best position the patient? A) In a high Fowlers position B) On the left side-lying position C) In a flat, supine position D) In the Trendelenburg position

: A Feedback: Proper positioning can help reduce venous return to the heart. The patient is positioned upright. If the patient is unable to sit with the lower extremities dependent, the patient may be placed in an upright position in bed. The supine position and Trendelenburg positions will not reduce venous return, lower the output of the right ventricle, or decrease lung congestion. Similarly, side-lying does not promote circulation.

23. The nurse is reviewing the medication administration record of a patient diagnosed with systolic HF. What medication should the nurse anticipate administering to this patient? A) A beta-adrenergic blocker - 574 B) An antiplatelet aggregator C) A calcium channel blocker D) A nonsteroidal anti-inflammatory drug (NSAID)

: A Feedback: Several medications are routinely prescribed for systolic HF, including ACE inhibitors, beta-blockers, diuretics, and digitalis. Calcium channel blockers, antiplatelet aggregators, and NSAIDs are not commonly prescribed.

26. The nurse is performing a physical assessment on a patient suspected of having HF. The presence of what sound would signal the possibility of impending HF? A) An S3 heart sound B) Pleural friction rub C) Faint breath sounds D) A heart murmur

: A Feedback: The heart is auscultated for an S3 heart sound, a sign that the heart is beginning to fail and that increased blood volume fills the ventricle with each beat. HF does not normally cause a pleural friction rub or murmurs. Changes in breath sounds occur, such as the emergence of crackles or wheezes, but faint breath sounds are less characteristic of HF.

37. A cardiac patients resistance to left ventricular filling has caused blood to back up into the patients circulatory system. What health problem is likely to result? A) Acute pulmonary edema B) Right-sided HF C) Right ventricular hypertrophy D) Left-sided HF

: A Feedback: With increased resistance to left ventricular filling, blood backs up into the pulmonary circulation. The patient quickly develops pulmonary edema from the blood volume overload in the lungs. When the blood backs up into the pulmonary circulation, right-sided HF, left-sided HF, and right ventricular hypertrophy do not directly occur.

33. The nurse is creating a care plan for a patient diagnosed with HF. When addressing the problem of anxiety, what interventions should the nurse include in the care plan? Select all that apply. A) Facilitate the presence of friends and family whenever possible. B) Teach the patient about the harmful effects of anxiety on cardiac function. C) Provide supplemental oxygen, as needed. D) Provide validation of the patients expressions of anxiety. E) Administer benzodiazepines two to three times daily.

: A, C, D Feedback: The nurse should empathically validate the patients sensations of anxiety. The presence of friends and family are frequently beneficial and oxygen supplementation promotes comfort. Antianxiety medications may be necessary for some patients, but alternative methods of relief should be prioritized. As well, medications are administered on a PRN basis. Teaching the patient about the potential harms of anxiety is likely to exacerbate, not relieve, the problem.

21. The nurse is planning the care of a patient with HF. The nurse should identify what overall goals of this patients care? - 573 A) Improve functional status B) Prevent endocarditis. C) Extend survival. D) Limit physical activity. E) Relieve patient symptoms.

: A, C, E Feedback: The overall goals of management of HF are to relieve the patients symptoms, to improve functional status and quality of life, and to extend survival. Activity limitations should be accommodated, but reducing activity is not a goal. Endocarditis is not a common complication of HF and preventing it is not a major goal of care.

16. The nurses comprehensive assessment of a patient who has HF includes evaluation of the patients hepatojugular reflux. What action should the nurse perform during this assessment? A) Elevate the patients head to 90 degrees. B) Press the right upper abdomen. C) Press above the patients symphysis pubis. D) Lay the patient flat in bed.

: B - 571 Feedback: Hepatojugular reflux, a sign of right-sided heart failure, is assessed with the head of the bed at a 45- degree angle. As the right upper abdomen (the area over the liver) is compressed for 30 to 40 seconds, the nurse observes the internal jugular vein. If the internal jugular vein becomes distended, a patient has positive hepatojugular reflux.

10. A patient presents to the ED complaining of increasing shortness of breath. The nurse assessing the patient notes a history of left-sided HF. The patient is agitated and occasionally coughing up pink-tinged, foamy sputum. The nurse should recognize the signs and symptoms of what health problem? A) Right-sided heart failure B) Acute pulmonary edema C) Pneumonia D) Cardiogenic shock

: B Feedback: Because of decreased contractility and increased fluid volume and pressure in patients with HF, fluid may be driven from the pulmonary capillary beds into the alveoli, causing pulmonary edema and signs and symptoms described. In right-sided heart failure, the patient exhibits hepatomegaly, jugular vein distention, and peripheral edema. In pneumonia, the patient would have a temperature spike, and sputum that varies in color. Cardiogenic shock would show signs of hypotension and tachycardia.

17. The nurse overseeing care in the ICU reviews the shift report on four patients. The nurse recognizes which patient to be at greatest risk for the development of cardiogenic shock? A) The patient admitted with acute renal failure B) The patient admitted following an MI C) The patient admitted with malignant hypertension D) The patient admitted following a stroke

: B Feedback: Cardiogenic shock may occur following an MI when a large area of the myocardium becomes ischemic, necrotic, and hypokinetic. It also can occur as a result of end-stage heart failure, cardiac tamponade, pulmonary embolism, cardiomyopathy, and dysrhythmias. While patients with acute renal failure are at risk for dysrhythmias and patients experiencing a stroke are at risk for thrombus formation, the patient admitted following an MI is at the greatest risk for development of cardiogenic shock when compared with the other listed diagnoses.

4. Which assessment would be most appropriate for a patient who is receiving a loop diuretic for HF? A) Monitor liver function studies B) Monitor for hypotension C) Assess the patients vitamin D intake D) Assess the patient for hyperkalemia

: B Feedback: Diuretic therapy increases urine output and decreases blood volume, which places the patient at risk of hypotension. Patients are at risk of losing potassium with loop diuretic therapy and need to continue with potassium in their diet; hypokalemia is a consequent risk. Liver function is rarely compromised by diuretic therapy and vitamin D intake is not relevant.

32. The nurse is addressing exercise and physical activity during discharge education with a patient diagnosed with HF. What should the nurse teach this patient about exercise? A) Do not exercise unsupervised. - 578 B) Eventually aim to work up to 30 minutes of exercise each day. C) Slow down if you get dizzy or short of breath. D) Start your exercise program with high-impact activities.

: B Feedback: Eventually, a total of 30 minutes of physical activity every day should be encouraged. Supervision is not necessarily required and the emergence of symptoms should prompt the patient to stop exercising, not simply to slow the pace. Low-impact activities should be prioritized.

31. The nurse is educating an 80-year-old patient diagnosed with HF about his medication regimen. What should the nurse to teach this patient about the use of oral diuretics? A) Avoid drinking fluids for 2 hours after taking the diuretic. B) Take the diuretic in the morning to avoid interfering with sleep. C) Avoid taking the medication within 2 hours consuming dairy products. D) Take the diuretic only on days when experiencing shortness of breath.

: B Feedback: Oral diuretics should be administered early in the morning so that diuresis does not interfere with the patients nighttime rest. Discussing the timing of medication administration is especially important for elderly patients who may have urinary urgency or incontinence. The nurse would not teach the patient about the timing of fluid intake. Fluid intake does not need to be adjusted and dairy products are not contraindicated.

5. The nurse is assessing a patient who is known to have right-sided HF. What assessment finding is most consistent with this patients diagnosis? A) Pulmonary edema - 566 B) Distended neck veins C) Dry cough D) Orthopnea

: B Feedback: Right-sided HF may manifest by distended neck veins, dependent edema, hepatomegaly, weight gain, ascites, anorexia, nausea, nocturia, and weakness. The other answers do not apply.

15. A patient with HF is placed on a low-sodium diet. Which statement by the patient indicates that the nurses nutritional teaching plan has been effective? A) I will have a ham and cheese sandwich for lunch. B) I will have a baked potato with broiled chicken for dinner. C) I will have a tossed salad with cheese and croutons for lunch. D) I will have chicken noodle soup with crackers and an apple for lunch.

: B Feedback: The patients choice of a baked potato with broiled chicken indicates that the teaching plan has been effective. Potatoes and chicken are relatively low in sodium. Ham, cheese, and soup are often high in sodium.

8. The nurse is caring for a 68-year-old patient the nurse suspects has digoxin toxicity. In addition to physical assessment, the nurse should collect what assessment datum? A) Skin turgor B) Potassium level C) White blood cell count D) Peripheral pulses

: B Feedback: The serum potassium level is monitored because the effect of digoxin is enhanced in the presence of hypokalemia and digoxin toxicity may occur. Skin turgor, white cell levels, and peripheral pulses are not normally affected in cases of digitalis toxicity.

34. The critical care nurse is caring for a patient who is in cardiogenic shock. What assessments must the nurse perform on this patient? Select all that apply. A) Platelet level B) Fluid status C) Cardiac rhythm - 579 D) Action of medications E) Sputum volume

: B, C, D Feedback: The critical care nurse must carefully assess the patient in cardiogenic shock, observe the cardiac rhythm, monitor hemodynamic parameters, monitor fluid status, and adjust medications and therapies based on the assessment data. Platelet levels and sputum production are not major assessment parameters in a patient who is experiencing cardiogenic shock.

24. The nurse is caring for a patient with systolic HF whose previous adverse reactions preclude the safe use of ACE inhibitors. The nurse should anticipate that the prescriber may choose what combination of drugs? A) Loop diuretic and antiplatelet aggregator B) Loop diuretic and calcium channel blocker C) Combination of hydralazine and isosorbide dinitrate D) Combination of digoxin and normal saline

: C Feedback: A combination of hydralazine and isosorbide dinitrate may be an alternative for patients who cannot take ACE inhibitors. Antiplatelet aggregators, calcium channel blockers, and normal saline are not typically prescribed.

39. Diagnostic imaging reveals that the quantity of fluid in a clients pericardial sac is dangerously increased. The nurse should collaborate with the other members of the care team to prevent the development of what complication? A) Pulmonary edema B) Pericardiocentesis C) Cardiac tamponade D) Pericarditis

: C Feedback: An increase in pericardial fluid raises the pressure within the pericardial sac and compresses the heart, eventually causing cardiac tamponade. Pericardiocentesis is the treatment for this complication. Pericarditis and pulmonary edema do not result from this pathophysiological process.

1. The nurse notes that a patient has developed a cough productive for mucoid sputum, is short of breath, has cyanotic hands, and has noisy, moist-sounding, rapid breathing. These symptoms and signs are suggestive of what health problem? A) Pericarditis B) Cardiomyopathy C) Pulmonary edema D) Right ventricular hypertrophy

: C Feedback: As a result of decreased cerebral oxygenation, the patient with pulmonary edema becomes increasingly restless and anxious. Along with a sudden onset of breathlessness and a sense of suffocation, the patients hands become cold and moist, the nail beds become cyanotic (bluish), and the skin turns ashen (gray). The pulse is weak and rapid, and the neck veins are distended. Incessant coughing may occur, producing increasing quantities of foamy sputum. Pericarditis, ventricular hypertrophy, and cardiomyopathy do not involve wet breath sounds or mucus production.

3. The triage nurse in the ED is assessing a patient with chronic HF who has presented with worsening symptoms. In reviewing the patients medical history, what is a potential primary cause of the patients heart failure? A) Endocarditis B) Pleural effusion C) Atherosclerosis D) Atrial-septal defect

: C Feedback: Atherosclerosis of the coronary arteries is the primary cause of HF. Pleural effusion, endocarditis, and an atrial-septal defect are not health problems that contribute to the etiology of HF.

14. The nurse is caring for an 84-year-old man who has just returned from the OR after inguinal hernia repair. The OR report indicates that the patient received large volumes of IV fluids during surgery and the nurse recognizes that the patient is at risk for left-sided heart failure. What signs and symptoms would indicate left-sided heart failure? A) Jugular vein distention B) Right upper quadrant pain - 570 C) Bibasilar fine crackles D) Dependent edema

: C Feedback: Bibasilar fine crackles are a sign of alveolar fluid, a sequela of left ventricular fluid, or pressure overload. Jugular vein distention, right upper quadrant pain (hepatomegaly), and dependent edema are caused by right-sided heart failure, usually a chronic condition.

13. The nurse is providing discharge education to a patient diagnosed with HF. What should the nurse teach this patient to do to assess her fluid balance in the home setting? A) Monitor her blood pressure daily B) Assess her radial pulses daily C) Monitor her weight daily D) Monitor her bowel movements

: C Feedback: To assess fluid balance at home, the patient should monitor daily weights at the same time every day. Assessing radial pulses and monitoring the blood pressure may be done, but these measurements do not provide information about fluid balance. Bowel function is not indicative of fluid balance.

27. An older adult patient with HF is being discharged home on an ACE inhibitor and a loop diuretic. The patients most recent vital signs prior to discharge include oxygen saturation of 93% on room air, heart rate of 81 beats per minute, and blood pressure of 94/59 mm Hg. When planning this patients subsequent care, what nursing diagnosis should be identified? A) Risk for ineffective tissue perfusion related to dysrhythmia B) Risk for fluid volume excess related to medication regimen C) Risk for ineffective breathing pattern related to hypoxia D) Risk for falls related to hypotension

: D Feedback: - 576 The combination of low BP, diuretic use, and ACE inhibitor use constitute a risk for falls. There is no evidence, or heightened risk, of dysrhythmia. The patients medications create a risk for fluid deficit, not fluid excess. Hypoxia is a risk for all patients with HF, but this is not in evidence for this patient at this time.

12. The nurse has entered a patients room and found the patient unresponsive and not breathing. What is the - 569 nurses next appropriate action? A) Palpate the patients carotid pulse. B) Illuminate the patients call light. C) Begin performing chest compressions. D) Activate the Emergency Response System (ERS).

: D Feedback: After checking for responsiveness and breathing, the nurse should activate the ERS. Assessment of carotid pulse should follow and chest compressions may be indicated. Illuminating the call light is an insufficient response.

9. The triage nurse in the ED is performing a rapid assessment of a man with complaints of severe chest pain and shortness of breath. The patient is diaphoretic, pale, and weak. When the patient collapses, what should the nurse do first? A) Check for a carotid pulse. B) Apply supplemental oxygen. C) Give two full breaths. D) Gently shake and shout, Are you OK?

: D Feedback: Assessing responsiveness is the first step in basic life support. Opening the airway and checking for respirations should occur next. If breathing is absent, two breaths should be given, usually accompanied by supplementary oxygen. Circulation is checked by palpating the carotid artery. - 568

19. The cardiac monitor alarm alerts the critical care nurse that the patient is showing no cardiac rhythm on the monitor. The nurses rapid assessment suggests cardiac arrest. In providing cardiac resuscitation documentation, how will the nurse describe this initial absence of cardiac rhythm? A) Pulseless electrical activity (PEA) B) Ventricular fibrillation C) Ventricular tachycardia D) Asystole

: D Feedback: Cardiac arrest occurs when the heart ceases to produce an effective pulse and circulate blood. It may be caused by a cardiac electrical event such as ventricular fibrillation, ventricular tachycardia, profound bradycardia, or when there is no heart rhythm at all (asystole). Cardiac arrest may also occur when electrical activity is present, but there is ineffective cardiac contraction or circulating volume, which is PEA. Asystole is the only condition that involves the absolute absence of a heart rhythm.

2. The nurse is assessing an older adult patient with numerous health problems. What assessment datum indicates an increase in the patients risk for heart failure (HF)? A) The patient takes Lasix (furosemide) 20 mg/day. B) The patients potassium level is 4.7 mEq/L. C) The patient is an African American man. D) The patients age is greater than 65.

: D Feedback: HF is the most common reason for hospitalization of people older than 65 years of age and is the second most common reason for visits to a physicians office. A potassium level of 4.7 mEq/L is within reference range and does not indicate an increased risk for HF. The fact that the patient takes Lasix 20 - 565 mg/day does not indicate an increased risk for HF, although this drug is often used in the treatment of HF. The patient being an African American man does not indicate an increased risk for HF.

20. The nurse is reviewing a newly admitted patients electronic health record, which notes a history of orthopnea? What nursing action is most clearly indicated? A) Teach the patient deep breathing and coughing exercises. B) Administer supplemental oxygen at all times. C) Limit the patients activity level. D) Avoid positioning the patient supine.

: D Feedback: Orthopnea is defined as difficulty breathing while lying flat. This is a possible complication of HF and, consequently, the nurse should avoid positioning the patient supine. Oxygen supplementation may or may not be necessary and activity does not always need to be curtailed. Deep breathing and coughing exercises do not directly address this symptom.

18. When assessing the patient with pericardial effusion, the nurse will assess for pulsus paradoxus. Pulsus paradoxus is characterized by what assessment finding? A) A diastolic blood pressure that is lower during exhalation B) A diastolic blood pressure that is higher during inhalation C) A systolic blood pressure that is higher during exhalation D) A systolic blood pressure that is lower during inhalation

: D Feedback: Systolic blood pressure that is markedly lower during inhalation is called pulsus paradoxus. The difference in systolic pressure between the point that is heard during exhalation and the point that is - 572 heard during inhalation is measured. Pulsus paradoxus exceeding 10 mm Hg is abnormal.

A nurse is creating an education plan for a patient with venous insufficiency. What measure should the nurse include in the plan? A) Avoiding tight-fitting socks. B) Limit activity whenever possible. C) Sleep with legs in a dependent position. D) Avoid the use of pressure stockings.

A

A patient is brought to the ED and determined to be experiencing symptomatic sinus bradycardia. The nurse caring for this patient is aware the medication of choice for treatment of this dysrhythmia is the administration of atropine. What guidelines will the nurse follow when administering atropine? A)Administer atropine 0.5 mg as an IV bolus every 3 to 5 minutes to a maximum of 3.0 mg. B)Administer atropine as a continuous infusion until symptoms resolve. C)Administer atropine as a continuous infusion to a maximum of 30 mg in 24 hours. D)Administer atropine 1.0 mg sublingually.

A

A patient who has undergone a femoral to popliteal bypass graft surgery returns to the surgical unit. Which assessments should the nurse perform during the first postoperative day? A) Assess pulse of affected extremity every 15 minutes at first. B) Palpate the affected leg for pain during every assessment. C) Assess the patient for signs and symptoms of compartment syndrome every 2 hours. D) Perform Doppler evaluation once daily.

A

The clinic nurse is caring for a 57-year-old client who reports experiencing leg pain whenever she walks several blocks. The patient has type 1 diabetes and has smoked a pack of cigarettes every day for the past 40 years. The physician diagnoses intermittent claudication. The nurse should provide what instruction about long-term care to the client? A) "Be sure to practice meticulous foot care." B) "Consider cutting down on your smoking." C) "Reduce your activity level to accommodate your limitations." D) "Try to make sure you eat enough protein."

A

The nurse is admitting a 32-year-old woman to the presurgical unit. The nurse learns during the admission assessment that the patient takes oral contraceptives. Consequently, the nurse's postoperative plan of care should include what intervention? A) Early ambulation and leg exercises B) Cessation of the oral contraceptives until 3 weeks postoperative C) Doppler ultrasound of peripheral circulation twice daily D) Dependent positioning of the patient's extremities when at rest

A

The nurse is caring for a patient who is in the recovery room following the implantation of an ICD. The patient has developed ventricular tachycardia (VT). What should the nurse assess and document? AECG to compare time of onset of VT and onset of device's shock B)ECG so physician can see what type of dysrhythmia the patient has C)Patient's level of consciousness (LOC) at the time of the dysrhythmia D)Patient's activity at time of dysrhythmia

A

The nurse is caring for a patient who returned from the tropics a few weeks ago and who sought care with signs and symptoms of lymphedema. The nurse's plan of care should prioritize what nursing diagnosis? A) Risk for infection related to lymphedema B) Disturbed body image related to lymphedema C) Ineffective health maintenance related to lymphedema D) Risk for deficient fluid volume related to lymphedema

A

The nurse is caring for an adult patient who has gone into ventricular fibrillation. When assisting with defibrillating the patient, what must the nurse do? A)Maintain firm contact between paddles and patient skin. B)Apply a layer of water as a conducting agent. C)Call "all clear" once before discharging the defibrillator. D)Ensure the defibrillator is in the sync mode.

A

When assessing venous disease in a patient's lower extremities, the nurse knows that what test will most likely be ordered? A) Duplex ultrasonography B) Echocardiography C) Positron emission tomography (PET) D) Radiography

A

maintain the pt on complete bedrest

A patient has been admitted to the ICU after being recently dx with an aneurysm and the patients admission orders include specific aneurysm precautions. What nursing action will the nurse incorporate into the patient plan of care?

place a pillow in the axilla when there is limited external rotation

A patient has had an ischemic stroke and has been admitted to the medical unit. What action should the nurse perform to best prevent joint deformities?

prepare to administer 3% NaCl by IV as ordered

After a subarachnoid hemorrhage, the patients lab results indicate a serum sodium level of less than 126 mEq/L. What is the nurses most appropriate action?

In preparation for cardiac surgery, a patient was taught about measures to prevent venous thromboembolism. What statement indicates that the patient clearly understood this education? A) "I'll try to stay in bed for the first few days to allow myself to heal." B) "I'll make sure that I don't cross my legs when I'm resting in bed." C) "I'll keep pillows under my knees to help my blood circulate better." D) "I'll put on those compression stockings if I get pain in my calves."

Ans: "I'll make sure that I don't cross my legs when I'm resting in bed." Feedback: To prevent venous thromboembolism, patients should avoid crossing the legs. Activity is generally begun as soon as possible and pillows should not be placed under the popliteal space. Compression stockings are often used to prevent venous thromboembolism, but they would not be applied when symptoms emerge.

2. A patient with primary hypertension comes to the clinic complaining of a gradual onset of blurry vision and decreased visual acuity over the past several weeks. The nurse is aware that these symptoms could be indicative of what? A) Retinal blood vessel damage B) Glaucoma C) Cranial nerve damage D) Hypertensive emergency

Ans: A Feedback: Blurred vision, spots in front of the eyes, and diminished visual acuity can mean retinal blood vessel damage indicative of damage elsewhere in the vascular system as a result of hypertension. Glaucoma and cranial nerve damage do not normally cause these symptoms. A hypertensive emergency would have a more rapid onset.

20. The nurse is developing a nursing care plan for a patient who is being treated for hypertension. What is a measurable patient outcome that the nurse should include? A) Patient will reduce Na+ intake to no more than 2.4 g daily. B) Patient will have a stable BUN and serum creatinine levels. C) Patient will abstain from fat intake and reduce calorie intake. D) Patient will maintain a normal body weight.

Ans: A Feedback: Dietary sodium intake of no more than 2.4 g sodium is recommended as a dietary lifestyle modification to prevent and manage hypertension. Giving a specific amount of allowable sodium intake makes this a measurable goal. None of the other listed goals is quantifiable and measurable.

14. A nurse is teaching an adult female patient about the risk factors for hypertension. What should the nurse explain as risk factors for primary hypertension? A) Obesity and high intake of sodium and saturated fat B) Diabetes and use of oral contraceptives C) Metabolic syndrome and smoking D) Renal disease and coarctation of the aorta

Ans: A Feedback: Obesity, stress, high intake of sodium or saturated fat, and family history are all risk factors for primary hypertension. Diabetes and oral contraceptives are risk factors for secondary hypertension. Metabolic syndrome, renal disease, and coarctation of the aorta are causes of secondary hypertension.

10. A patient with primary hypertension complains of dizziness with ambulation. The patient is currently on an alpha-adrenergic blocker and the nurse assesses characteristic signs and symptoms of postural hypotension. When teaching this patient about risks associated with postural hypotension, what should the nurse emphasize? A) Rising slowly from a lying or sitting position B) Increasing fluids to maintain BP C) Stopping medication if dizziness persists D) Taking medication first thing in the morning

Ans: A Feedback: Patients who experience postural hypotension should be taught to rise slowly from a lying or sitting position and use a cane or walker if necessary for safety. It is not necessary to teach these patients about increasing fluids or taking medication in the morning (this would increase the effects of dizziness). Patient should not be taught to stop the medication if dizziness persists because this is unsafe and beyond the nurse's scope of practice.

18. A patient has come to the clinic for a follow-up assessment that will include a BP reading. To ensure an accurate reading, the nurse should confirm that the patient has done which of the following? A) Tried to rest quietly for 5 minutes before the reading is taken B) Refrained from smoking for at least 8 hours C) Drunk adequate fluids during the day prior D) Avoided drinking coffee for 12 hours before the visit

Ans: A Feedback: Prior to the nurse assessing the patient's BP, the patient should try to rest quietly for 5 minutes. The forearm should be positioned at heart level. Caffeine products and cigarette smoking should be avoided for at least 30 minutes prior to the visit. Recent fluid intake is not normally relevant.

16. A patient with secondary hypertension has come into the clinic for a routine check-up. The nurse is aware that the difference between primary hypertension and secondary hypertension is which of the following? A) Secondary hypertension has a specific cause. B) Secondary hypertension has a more gradual onset than primary hypertension. C) Secondary hypertension does not cause target organ damage. D) Secondary hypertension does not normally respond to antihypertensive drug therapy.

Ans: A Feedback: Secondary hypertension has a specific identified cause. A cause could include narrowing of the renal arteries, renal parenchymal disease, hyperaldosteronism, certain medications, pregnancy, and coarctation of the aorta. Secondary hypertension does respond to antihypertensive drug therapy and can cause target organ damage if left untreated.

7. A patient has been prescribed antihypertensives. After assessment and analysis, the nurse has identified a nursing diagnosis of risk for ineffective health maintenance related to nonadherence to therapeutic regimen. When planning this patient's care, what desired outcome should the nurse identify? A) Patient takes medication as prescribed and reports any adverse effects. B) Patient's BP remains consistently below 140/90 mm Hg. C) Patient denies signs and symptoms of hypertensive urgency. D) Patient is able to describe modifiable risk factors for hypertension.

Ans: A Feedback: The most appropriate expected outcome for a patient who is given the nursing diagnosis of risk for ineffective health maintenance is that he or she takes the medication as prescribed. The other listed goals are valid aspects of care, but none directly relates to the patient's role in his or her treatment regimen.

9. A patient newly diagnosed with hypertension asks the nurse what happens when uncontrolled hypertension is prolonged. The nurse explains that a patient with prolonged, uncontrolled hypertension is at risk for developing what health problem? A) Renal failure B) Right ventricular hypertrophy C) Glaucoma D) Anemia

Ans: A Feedback: When uncontrolled hypertension is prolonged, it can result in renal failure, myocardial infarction, stroke, impaired vision, left ventricular hypertrophy, and cardiac failure. Glaucoma and anemia are not directly associated with hypertension.

A nurse is working with a patient who has been scheduled for a percutaneous coronary intervention (PCI) later in the week. What anticipatory guidance should the nurse provide to the patient? A) He will remain on bed rest for 48 to 72 hours after the procedure. B) He will be given vitamin K infusions to prevent bleeding following PCI. C) A sheath will be placed over the insertion site after the procedure is finished. D) The procedure will likely be repeated in 6 to 8 weeks to ensure success.

Ans: A sheath will be placed over the insertion site after the procedure is finished. Feedback: A sheath is placed over the PCI access site and kept in place until adequate coagulation is achieved. Patients resume activity a few hours after PCI and repeated treatments may or may not be necessary. Anticoagulants, not vitamin K, are administered during PCI.

23. The nurse is teaching a patient about some of the health consequences of uncontrolled hypertension. What health problems should the nurse describe? Select all that apply. A) Transient ischemic attacks B) Cerebrovascular accident C) Retinal hemorrhage D) Venous insufficiency E) Right ventricular hypertrophy

Ans: A, B, C Feedback: Potential complications of hypertension include the following: left ventricular hypertrophy; MI; heart failure; transient ischemic attacks (TIAs); cerebrovascular accident; renal insufficiency and failure; and retinal hemorrhage. Venous insufficiency and right ventricular hypertrophy are not potential complications of uncontrolled hypertension.

The nurse providing care for a patient post PTCA knows to monitor the patient closely. For what complications should the nurse monitor the patient? Select all that apply. A) Abrupt closure of the coronary artery B) Venous insufficiency C) Bleeding at the insertion site D) Retroperitoneal bleeding E) Arterial occlusion

Ans: Abrupt closure of the coronary artery, Bleeding at the insertion site, Retroperitoneal bleeding, Arterial occlusion Feedback: Complications after the procedure may include abrupt closure of the coronary artery and vascular complications, such as bleeding at the insertion site, retroperitoneal bleeding, hematoma, and arterial occlusion, as well as acute renal failure. Venous insufficiency is not a postprocedure complication of a PTCA.

The nurse is creating a plan of care for a patient with acute coronary syndrome. What nursing action should be included in the patient's care plan? A) Facilitate daily arterial blood gas (ABG) sampling. B) Administer supplementary oxygen, as needed. C) Have patient maintain supine positioning when in bed. D) Perform chest physiotherapy, as indicated.

Ans: Administer supplementary oxygen, as needed. Feedback: Oxygen should be administered along with medication therapy to assist with symptom relief. Administration of oxygen raises the circulating level of oxygen to reduce pain associated with low levels of myocardial oxygen. Physical rest in bed with the head of the bed elevated or in a supportive chair helps decrease chest discomfort and dyspnea. ABGs are diagnostic, not therapeutic, and they are rarely needed on a daily basis. Chest physiotherapy is not used in the treatment of ACS.

You are writing a care plan for a patient who has been diagnosed with angina pectoris. The patient describes herself as being "distressed" and "shocked" by her new diagnosis. What nursing diagnosis is most clearly suggested by the woman's statement? A) Spiritual distress related to change in health status B) Acute confusion related to prognosis for recovery C) Anxiety related to cardiac symptoms D) Deficient knowledge related to treatment of angina pectoris

Ans: Anxiety related to cardiac symptoms Feedback: Although further assessment is warranted, it is not unlikely that the patient is experiencing anxiety. In patients with CAD, this often relates to the threat of sudden death. There is no evidence of confusion (i.e., delirium or dementia) and there may or may not be a spiritual element to her concerns. Similarly, it is not clear that a lack of knowledge or information is the root of her anxiety.

8. The nurse is providing care for a patient with a new diagnosis of hypertension. How can the nurse best promote the patient's adherence to the prescribed therapeutic regimen? A) Screen the patient for visual disturbances regularly. B) Have the patient participate in monitoring his or her own BP. C) Emphasize the dire health outcomes associated with inadequate BP control. D) Encourage the patient to lose weight and exercise regularly.

Ans: B Feedback: Adherence to the therapeutic regimen increases when patients actively participate in self-care, including self-monitoring of BP and diet. Dire warnings may motivate some patients, but for many patients this is not an appropriate or effective strategy. Screening for vision changes and promoting healthy lifestyle are appropriate nursing actions, but do not necessarily promote adherence to a therapeutic regimen.

13. The nursing lab instructor is teaching student nurses how to take blood pressure. To ensure accurate measurement, the lab instructor would teach the students to avoid which of the following actions? A) Measuring the BP after the patient has been seated quietly for more than 5 minutes B) Taking the BP at least 10 minutes after nicotine or coffee ingestion C) Using a cuff with a bladder that encircles at least 80% of the limb D) Using a bare forearm supported at heart level on a firm surface

Ans: B Feedback: Blood pressures should be taken with the patient seated with arm bare, supported, and at heart level. The patient should not have smoked tobacco or taken caffeine in the 30 minutes preceding the measurement. The patient should rest quietly for 5 minutes before the reading is taken. The cuff bladder should encircle at least 80% of the limb being measured and have a width of at least 40% of limb circumference. Using a cuff that is too large results in a lower BP and a cuff that is too small will give a higher BP measurement.

22. The hospital nurse cares for many patients who have hypertension. What nursing diagnosis is most common among patients who are being treated for this health problem? A) Deficient knowledge regarding the lifestyle modifications for management of hypertension B) Noncompliance with therapeutic regimen related to adverse effects of prescribed therapy C) Deficient knowledge regarding BP monitoring D) Noncompliance with treatment regimen related to medication costs

Ans: B Feedback: Deviation from the therapeutic program is a significant problem for people with hypertension and other chronic conditions requiring lifetime management. For many patients, this is related to adverse effects of medications. Medication cost is relevant for many patients, but adverse effects are thought to be a more significant barrier. Many patients are aware of necessary lifestyle modification, but do not adhere to them. Most patients are aware of the need to monitor their BP.

15. The nurse is caring for an older adult with a diagnosis of hypertension who is being treated with a diuretic and beta-blocker. Which of the following should the nurse integrate into the management of this client's hypertension? A) Ensure that the patient receives a larger initial dose of antihypertensive medication due to impaired absorption. B) Pay close attention to hydration status because of increased sensitivity to extracellular volume depletion. C) Recognize that an older adult is less likely to adhere to his or her medication regimen than a younger patient. D) Carefully assess for weight loss because of impaired kidney function resulting from normal aging.

Ans: B Feedback: Elderly people have impaired cardiovascular reflexes and thus are more sensitive to extracellular volume depletion caused by diuretics. The nurse needs to assess hydration status, low BP, and postural hypotension carefully. Older adults may have impaired absorption, but they do not need a higher initial dose of an antihypertensive than a younger person. Adherence to treatment is not necessarily linked to age. Kidney function and absorption decline with age; less, rather than more antihypertensive medication is prescribed. Weight gain is not necessarily indicative of kidney function decline.

6. A 40-year-old male newly diagnosed with hypertension is discussing risk factors with the nurse. The nurse talks about lifestyle changes with the patient and advises that the patient should avoid tobacco use. What is the primary rationale behind that advice to the patient? A) Quitting smoking will cause the patient's hypertension to resolve. B) Tobacco use increases the patient's concurrent risk of heart disease. C) Tobacco use is associated with a sedentary lifestyle. D) Tobacco use causes ventricular hypertrophy.

Ans: B Feedback: Smoking increases the risk for heart disease, for which a patient with hypertension is already at an increased risk. Quitting will not necessarily cause hypertension to resolve and smoking does not directly cause ventricular hypertrophy. The association with a sedentary lifestyle is true, but this is not the main rationale for the nurse's advice; the association with heart disease is more salient.

29. A patient has been diagnosed as being prehypertensive. What should the nurse encourage this patient to do to aid in preventing a progression to a hypertensive state? A) Avoid excessive potassium intake. B) Exercise on a regular basis. C) Eat less protein and more vegetables. D) Limit morning activity.

Ans: B Feedback: To prevent or delay progression to hypertension and reduce risk, JNC 7 urged health care providers to encourage people with blood pressures in the prehypertension category to begin lifestyle modifications, such as nutritional changes and exercise. There is no need for patients to limit their activity in the morning or to avoid potassium and protein intake.

36. The nurse is reviewing the medication administration record of a patient who takes a variety of medications for the treatment of hypertension. What potential therapeutic benefits of antihypertensives should the nurse identify? Select all that apply. A) Increased venous return B) Decreased peripheral resistance C) Decreased blood volume D) Decreased strength and rate of myocardial contractions E) Decreased blood viscosity

Ans: B, C, D Feedback: The medications used for treating hypertension decrease peripheral resistance, blood volume, or the strength and rate of myocardial contraction. Antihypertensive medications do not increase venous return or decrease blood viscosity.

The nurse is participating in the care conference for a patient with ACS. What goal should guide the care team's selection of assessments, interventions, and treatments? A) Maximizing cardiac output while minimizing heart rate B) Decreasing energy expenditure of the myocardium C) Balancing myocardial oxygen supply with demand D) Increasing the size of the myocardial muscle

Ans: Balancing myocardial oxygen supply with demand Feedback: Balancing myocardial oxygen supply with demand (e.g., as evidenced by the relief of chest pain) is the top priority in the care of the patient with ACS. Treatment is not aimed directly at minimizing heart rate because some patients experience bradycardia. Increasing the size of the myocardium is never a goal. Reducing the myocardium's energy expenditure is often beneficial, but this must be balanced with productivity.

A 48-year-old man presents to the ED complaining of severe substernal chest pain radiating down his left arm. He is admitted to the coronary care unit (CCU) with a diagnosis of myocardial infarction (MI). What nursing assessment activity is a priority on admission to the CCU? A) Begin ECG monitoring. B) Obtain information about family history of heart disease. C) Auscultate lung fields. D) Determine if the patient smokes.

Ans: Begin ECG monitoring. Feedback: The 12-lead ECG provides information that assists in ruling out or diagnosing an acute MI. It should be obtained within 10 minutes from the time a patient reports pain or arrives in the ED. By monitoring serial ECG changes over time, the location, evolution, and resolution of an MI can be identified and monitored; life-threatening arrhythmias are the leading cause of death in the first hours after an MI. Obtaining information about family history of heart disease and whether the patient smokes are not immediate priorities in the acute phase of MI. Data may be obtained from family members later. Lung fields are auscultated after oxygenation and pain control needs are met.

A patient with an occluded coronary artery is admitted and has an emergency percutaneous transluminal coronary angioplasty (PTCA). The patient is admitted to the cardiac critical care unit after the PTCA. For what complication should the nurse most closely monitor the patient? A) Hyperlipidemia B) Bleeding at insertion site C) Left ventricular hypertrophy D) Congestive heart failure

Ans: Bleeding at insertion site Feedback: Complications of PTCA may include bleeding at the insertion site, abrupt closure of the artery, arterial thrombosis, and perforation of the artery. Complications do not include hyperlipidemia, left ventricular hypertrophy, or congestive heart failure; each of these problems takes an extended time to develop and none is emergent.

17. The nurse is assessing a patient new to the clinic. Records brought to the clinic with the patient show the patient has hypertension and that her current BP readings approximate the readings from when she was first diagnosed. What contributing factor should the nurse first explore in an effort to identify the cause of the client's inadequate BP control? A) Progressive target organ damage B) Possibility of medication interactions C) Lack of adherence to prescribed drug therapy D) Possible heavy alcohol use or use of recreational drugs

Ans: C Feedback: Deviation from the therapeutic program is a significant problem for people with hypertension and other chronic conditions requiring lifetime management. An estimated 50% of patients discontinue their medications within 1 year of beginning to take them. Consequently, this is a more likely problem than substance use, organ damage, or adverse drug interactions.

1. An older adult is newly diagnosed with primary hypertension and has just been started on a beta-blocker. The nurse's health education should include which of the following? A) Increasing fluids to avoid extracellular volume depletion from the diuretic effect of the beta-blocker B) Maintaining a diet high in dairy to increase protein necessary to prevent organ damage C) Use of strategies to prevent falls stemming from postural hypotension D) Limiting exercise to avoid injury that can be caused by increased intracranial pressure

Ans: C Feedback: Elderly people have impaired cardiovascular reflexes and are more sensitive to postural hypotension. The nurse teaches patients to change positions slowly when moving from lying or sitting positions to a standing position, and counsels elderly patients to use supportive devices as necessary to prevent falls that could result from dizziness. Lifestyle changes, such as regular physical activity/exercise, and a diet rich in fruits, vegetables, and low-fat dairy products, is strongly recommended. Increasing fluids in elderly patients may be contraindicated due to cardiovascular disease. Increased intracranial pressure is not a risk and activity should not normally be limited.

5. A group of student nurses are practicing taking blood pressure. A 56-year-old male student has a blood pressure reading of 146/96 mm Hg. Upon hearing the reading, he exclaims, "My pressure has never been this high. Do you think my doctor will prescribe medication to reduce it?" Which of the following responses by the nursing instructor would be best? A) "Yes. Hypertension is prevalent among men; it is fortunate we caught this during your routine examination." B) "We will need to reevaluate your blood pressure because your age places you at high risk for hypertension." C) "A single elevated blood pressure does not confirm hypertension. You will need to have your blood pressure reassessed several times before a diagnosis can be made." D) "You have no need to worry. Your pressure is probably elevated because you are being tested."

Ans: C Feedback: Hypertension is confirmed by two or more readings with systolic pressure of at least 140 mm Hg and diastolic pressure of at least 90 mm Hg. An age of 56 does not constitute a risk factor in and of itself. The nurse should not tell the student that there is no need to worry.

12. A patient in a hypertensive emergency is admitted to the ICU. The nurse anticipates that the patient will be treated with IV vasodilators, and that the primary goal of treatment is what? A) Lower the BP to reduce onset of neurologic symptoms, such as headache and vision changes. B) Decrease the BP to a normal level based on the patient's age. C) Decrease the mean arterial pressure between 20% and 25% in the first hour of treatment. D) Reduce the BP to 120/75 mm Hg as quickly as possible.

Ans: C Feedback: Initially, the treatment goal in hypertensive emergencies is to reduce the mean arterial pressure by 25% in the first hour of treatment, with further reduction over the next 24 hours. Lowering the BP too fast may cause hypotension in a patient whose body has adjusted to hypertension and could cause a stroke, MI, or visual changes. Neurologic symptoms should be addressed, but this is not the primary focus of treatment planning.

24. The nurse is collaborating with the dietitian and a patient with hypertension to plan dietary modifications. These modifications should include which of the following? A) Reduced intake of protein and carbohydrates B) Increased intake of calcium and vitamin D C) Reduced intake of fat and sodium D) Increased intake of potassium, vitamin B12 and vitamin D

Ans: C Feedback: Lifestyle modifications usually include restricting sodium and fat intake, increasing intake of fruits and vegetables, and implementing regular physical activity. There is no need to increase calcium, potassium, and vitamin intake. Calorie restriction may be required for some patients, but a specific reduction in protein and carbohydrates is not normally indicated.

34. A student nurse is taking care of an elderly patient with hypertension during a clinical experience. The instructor asks the student about the relationships between BP and age. What would be the best answer by the student? A) "Because of reduced smooth muscle tone in blood vessels, blood pressure tends to go down with age, not up." B) "Decreases in the strength of arteries and the presence of venous insufficiency cause hypertension in the elderly." C) "Structural and functional changes in the cardiovascular system that occur with age contribute to increases in blood pressure." D) "The neurologic system of older adults is less efficient at monitoring and regulating blood pressure."

Ans: C Feedback: Structural and functional changes in the heart and blood vessels contribute to increases in BP that occur with aging. Venous insufficiency does not cause hypertension, however. Increased BP is not primarily a result of neurologic changes.

25. The critical care nurse is caring for a patient just admitted in a hypertensive emergency. The nurse should anticipate the administration of what medication? A) Warfarin (Coumadin) B) Furosemide (Lasix) C) Sodium nitroprusside (Nitropress) D) Ramipril (Altace)

Ans: C Feedback: The medications of choice in hypertensive emergencies are those that have an immediate effect. IV vasodilators, including sodium nitroprusside (Nitropress), nicardipine hydrochloride (Cardene), clevidipine (Cleviprex), fenoldopam mesylate (Corlopam), enalaprilat, and nitroglycerin, have immediate actions that are short lived (minutes to 4 hours), and they are therefore used for initial treatment. Ramipril is administered orally and would not meet the patient's immediate need for BP management. Diuretics, such as Lasix, are not used as initial treatments and there is no indication for anticoagulants such as Coumadin.

37. A newly diagnosed patient with hypertension is prescribed Diuril, a thiazide diuretic. What patient education should the nurse provide to this patient? A) "Eat a banana every day because Diuril causes moderate hyperkalemia." B) "Take over-the-counter potassium pills because Diuril causes your kidneys to lose potassium." C) "Diuril can cause low blood pressure and dizziness, especially when you get up suddenly." D) "Diuril increases sodium levels in your blood, so cut down on your salt."

Ans: C Feedback: Thiazide diuretics can cause postural hypotension, which may be potentiated by alcohol, barbiturates, opioids, or hot weather. Diuril does not cause either moderate hyperkalemia or severe hypokalemia and it does not result in hypernatremia.

3. A nurse is performing blood pressure screenings at a local health fair. While obtaining subjective assessment data from a patient with hypertension, the nurse learns that the patient has a family history of hypertension and she herself has high cholesterol and lipid levels. The patient says she smokes one pack of cigarettes daily and drinks "about a pack of beer" every day. The nurse notes what nonmodifiable risk factor for hypertension? A) Hyperlipidemia B) Excessive alcohol intake C) A family history of hypertension D) Closer adherence to medical regimen

Ans: C Feedback: Unlike cholesterol levels, alcohol intake and adherence to treatment, family history is not modifiable.

A patient in the cardiac step-down unit has begun bleeding from the percutaneous coronary intervention (PCI) access site in her femoral region. What is the nurse's most appropriate action? A) Call for assistance and initiate cardiopulmonary resuscitation. B) Reposition the patient's leg in a nondependent position. C) Promptly remove the femoral sheath. D) Call for help and apply pressure to the access site.

Ans: Call for help and apply pressure to the access site. Feedback: The femoral sheath produces pressure on the access site. Pressure will temporarily reduce bleeding and allow for subsequent interventions. Removing the sheath would exacerbate bleeding and repositioning would not halt it. CPR is not indicated unless there is evidence of respiratory or cardiac arrest.

The OR nurse is explaining to a patient that cardiac surgery requires the absence of blood from the surgical field. At the same time, it is imperative to maintain perfusion of body organs and tissues. What technique for achieving these simultaneous goals should the nurse describe? A) Coronary artery bypass graft (CABG) B) Percutaneous transluminal coronary angioplasty (PTCA) C) Atherectomy D) Cardiopulmonary bypass

Ans: Cardiopulmonary bypass Feedback: Cardiopulmonary bypass is often used to circulate and oxygenate blood mechanically while bypassing the heart and lungs. PTCA, atherectomy, and CABG are all surgical procedures, none of which achieves the two goals listed.

A nurse has taken on the care of a patient who had a coronary artery stent placed yesterday. When reviewing the patient's daily medication administration record, the nurse should anticipate administering what drug? A) Ibuprofen B) Clopidogrel C) Dipyridamole D) Acetaminophen

Ans: Clopidogrel Feedback: Because of the risk of thrombus formation within the stent, the patient receives antiplatelet medications, usually aspirin and clopidogrel. Ibuprofen and acetaminophen are not antiplatelet drugs. Dipyridamole is not the drug of choice following stent placement.

A patient who is postoperative day 1 following a CABG has produced 20 mL of urine in the past 3 hours and the nurse has confirmed the patency of the urinary catheter. What is the nurse's most appropriate action? A) Document the patient's low urine output and monitor closely for the next several hours. B) Contact the dietitian and suggest the need for increased oral fluid intake. C) Contact the patient's physician and suggest assessment of fluid balance and renal function. D) Increase the infusion rate of the patient's IV fluid to prompt an increase in renal function.

Ans: Contact the patient's physician and suggest assessment of fluid balance and renal function. Feedback: Nursing management includes accurate measurement of urine output. An output of less than 1 mL/kg/h may indicate hypovolemia or renal insufficiency. Prompt referral is necessary. IV fluid replacement may be indicated, but is beyond the independent scope of the dietitian or nurse.

A patient presents to the walk-in clinic complaining of intermittent chest pain on exertion, which is eventually attributed to angina. The nurse should inform the patient that angina is most often attributable to what cause? A) Decreased cardiac output B) Decreased cardiac contractility C) Infarction of the myocardium D) Coronary arteriosclerosis

Ans: Coronary arteriosclerosis Feedback: In most cases, angina pectoris is due to arteriosclerosis. The disease is not a result of impaired cardiac output or contractility. Infarction may result from untreated angina, but it is not a cause of the disease.

31. A community health nurse teaching a group of adults about preventing and treating hypertension. The nurse should encourage these participants to collaborate with their primary care providers and regularly monitor which of the following? A) Heart rate B) Sodium levels C) Potassium levels D) Blood lipid levels

Ans: D Feedback: Hypertension often accompanies other risk factors for atherosclerotic heart disease, such as dyslipidemia (abnormal blood fat levels), obesity, diabetes, metabolic syndrome, and a sedentary lifestyle. Individuals with hypertension need to monitor their sodium intake, but hypernatremia is not a risk factor for hypertension. In many patients, heart rate does not correlate closely with BP. Potassium levels do not normally relate to BP.

35. A 55-year-old patient comes to the clinic for a routine check-up. The patient's BP is 159/100 mm Hg and the physician diagnoses hypertension after referring to previous readings. The patient asks why it is important to treat hypertension. What would be the nurse's best response? A) "Hypertension can cause you to develop dangerous blood clots in your legs that can migrate to your lungs." B) "Hypertension puts you at increased risk of type 1 diabetes and cancer in your age group." C) "Hypertension is the leading cause of death in people your age." D) "Hypertension greatly increases your risk of stroke and heart disease."

Ans: D Feedback: Hypertension, particularly elevated systolic BP, increases the risk of death, stroke, and heart failure in people older than 50 years. Hypertension is not a direct precursor to pulmonary emboli, and it does not put older adults at increased risk of type 1 diabetes or cancer. It is not the leading cause of death in people 55 years of age.

4. The staff educator is teaching ED nurses about hypertensive crisis. The nurse educator should explain that hypertensive urgency differs from hypertensive emergency in what way? A) The BP is always higher in a hypertensive emergency. B) Vigilant hemodynamic monitoring is required during treatment of hypertensive emergencies. C) Hypertensive urgency is treated with rest and benzodiazepines to lower BP. D) Hypertensive emergencies are associated with evidence of target organ damage.

Ans: D Feedback: Hypertensive emergencies are acute, life-threatening BP elevations that require prompt treatment in an intensive care setting because of the serious target organ damage that may occur. Blood pressures are extremely elevated in both urgency and emergencies, but there is no evidence of target organ damage in hypertensive urgency. Extremely close hemodynamic monitoring of the patient's BP is required in both situations. The medications of choice in hypertensive emergencies are those with an immediate effect, such as IV vasodilators. Oral doses of fast-acting agents, such as beta-adrenergic blocking agents, angiotensin-converting enzyme inhibitors, or alpha-agonists, are recommended for the treatment of hypertensive urgencies.

26. A patient in hypertensive emergency is being cared for in the ICU. The patient has become hypovolemic secondary to natriuresis. What is the nurse's most appropriate action? A) Add sodium to the patient's IV fluid, as ordered. B) Administer a vasoconstrictor, as ordered. C) Promptly cease antihypertensive therapy. D) Administer normal saline IV, as ordered.

Ans: D Feedback: If there is volume depletion secondary to natriuresis caused by the elevated BP, then volume replacement with normal saline can prevent large, sudden drops in BP when antihypertensive medications are administered. Sodium administration, cessation of antihypertensive therapy, and administration of vasoconstrictors are not normally indicated.

30. The nurse is screening a number of adults for hypertension. What range of blood pressure is considered normal? A) Less than 140/90 mm Hg B) Less than 130/90 mm Hg C) Less than 129/89 mm Hg D) Less than 120/80 mm Hg

Ans: D Feedback: JNC 7 defines a blood pressure of less than 120/80 mm Hg as normal, 120 to 129/80 to 89 mm Hg as prehypertension, and 140/90 mm Hg or higher as hypertension.

28. A patient comes to the walk-in clinic complaining of frequent headaches. While assessing the patient's vital signs, the nurse notes the BP is 161/101 mm Hg. According to JNC 7, how would this patient's BP be defined if a similar reading were obtained at a subsequent office visit? A) High normal B) Normal C) Stage 1 hypertensive D) Stage 2 hypertensive

Ans: D Feedback: JNC 7 defines stage 2 hypertension as a reading 160/100 mm Hg.

40. A patient's recently elevated BP has prompted the primary care provider to prescribe furosemide (Lasix). The nurse should closely monitor which of the following? A) The client's oxygen saturation level B) The patient's red blood cells, hematocrit, and hemoglobin C) The patient's level of consciousness D) The patient's potassium level

Ans: D Feedback: Loop diuretics can cause potassium depletion. They do not normally affect level of consciousness, erythrocytes, or oxygen saturation.

32. A community health nurse is planning an educational campaign addressing hypertension. The nurse should anticipate that the incidence and prevalence of hypertension are likely to be highest among members of what ethnic group? A) Pacific Islanders B) African Americans C) Asian-Americans D) Hispanics

Ans: D Feedback: The prevalence of uncontrolled hypertension varies by ethnicity, with Hispanics and African Americans having the highest prevalence at approximately 63% and 57%, respectively.

33. The home health nurse is caring for a patient who has a comorbidity of hypertension. What assessment question most directly addresses the possibility of worsening hypertension? A) "Are you eating less salt in your diet?" B) "How is your energy level these days?" C) "Do you ever get chest pain when you exercise?" D) "Do you ever see spots in front of your eyes?"

Ans: D Feedback: To identify complications or worsening hypertension, the patient is questioned about blurred vision, spots in front of the eyes, and diminished visual acuity. The heart, nervous system, and kidneys are also carefully assessed, but angina pain and decreased energy are not normally suggestive of worsening hypertension. Sodium limitation is a beneficial lifestyle modification, but nonadherence to this is not necessarily a sign of worsening symptoms.

21. A patient with newly diagnosed hypertension has come to the clinic for a follow-up visit. The patient asks the nurse why she has to come in so often. What would be the nurse's best response? A) "We do this so you don't suffer a stroke." B) "We do this to determine how your blood pressure changes throughout the day." C) "We do this to see how often you should change your medication dose." D) "We do this to make sure your health is stable. We'll then monitor it at routinely scheduled intervals."

Ans: D Feedback: When hypertension is initially detected, nursing assessment involves carefully monitoring the BP at frequent intervals and then at routinely scheduled intervals. The reference to stroke is frightening and does not capture the overall rationale for the monitoring regimen. Changes throughout the day are not a clinical priority for most patients. The patient must not change his or her medication doses unilaterally.

The public health nurse is participating in a health fair and interviews a patient with a history of hypertension, who is currently smoking one pack of cigarettes per day. She denies any of the most common manifestations of CAD. Based on these data, the nurse would expect the focuses of CAD treatment most likely to be which of the following? A) Drug therapy and smoking cessation B) Diet and drug therapy C) Diet therapy only D) Diet therapy and smoking cessation

Ans: Diet therapy and smoking cessation Feedback: Due to the absence of symptoms, dietary therapy would likely be selected as the first-line treatment for possible CAD. Drug therapy would be determined based on a number of considerations and diagnostics findings, but would not be directly indicated. Smoking cessation is always indicated, regardless of the presence or absence of symptoms.

The nurse working on the coronary care unit is caring for a patient with ACS. How can the nurse best meet the patient's psychosocial needs? A) Reinforce the fact that treatment will be successful. B) Facilitate a referral to a chaplain or spiritual leader. C) Increase the patient's participation in rehabilitation activities. D) Directly address the patient's anxieties and fears.

Ans: Directly address the patient's anxieties and fears. Feedback: Alleviating anxiety and decreasing fear are important nursing functions that reduce the sympathetic stress response. Referrals to spiritual care may or may not be appropriate, and this does not relieve the nurse of responsibility for addressing the patient's psychosocial needs. Treatment is not always successful, and false hope should never be fostered. Participation in rehabilitation may alleviate anxiety for some patients, but it may exacerbate it for others.

The nurse is assessing a patient with acute coronary syndrome (ACS). The nurse includes a careful history in the assessment, especially with regard to signs and symptoms. What signs and symptoms are suggestive of ACS? Select all that apply. A) Dyspnea B) Unusual fatigue C) Hypotension D) Syncope E) Peripheral cyanosis

Ans: Dyspnea, Unusual fatigue, Syncope Feedback: Systematic assessment includes a careful history, particularly as it relates to symptoms: chest pain or discomfort, difficulty breathing (dyspnea), palpitations, unusual fatigue, faintness (syncope), or sweating (diaphoresis). Each symptom must be evaluated with regard to time, duration, and the factors that precipitate the symptom and relieve it, and in comparison with previous symptoms. Hypotension and peripheral cyanosis are not typically associated with ACS.

When discussing angina pectoris secondary to atherosclerotic disease with a patient, the patient asks why he tends to experience chest pain when he exerts himself. The nurse should describe which of the following phenomena? A) Exercise increases the heart's oxygen demands. B) Exercise causes vasoconstriction of the coronary arteries. C) Exercise shunts blood flow from the heart to the mesenteric area. D) Exercise increases the metabolism of cardiac medications.

Ans: Exercise increases the heart's oxygen demands. Feedback: Physical exertion increases the myocardial oxygen demand. If the patient has arteriosclerosis of the coronary arteries, then blood supply is diminished to the myocardium. Exercise does not cause vasoconstriction or interfere with drug metabolism. Exercise does not shunt blood flow away from the heart.

The nurse has just admitted a 66-year-old patient for cardiac surgery. The patient tearfully admits to the nurse that she is afraid of dying while undergoing the surgery. What is the nurse's best response? A) Explore the factors underlying the patient's anxiety. B) Teach the patient guided imagery techniques. C) Obtain an order for a PRN benzodiazepine. D) Describe the procedure in greater detail.

Ans: Explore the factors underlying the patient's anxiety. Feedback: An assessment of anxiety levels is required in the patient to assist the patient in identifying fears and developing coping mechanisms for those fears. The nurse must further assess and explore the patient's anxiety before providing interventions such as education or medications.

An OR nurse is preparing to assist with a coronary artery bypass graft (CABG). The OR nurse knows that the vessel most commonly used as source for a CABG is what? A) Brachial artery B) Brachial vein C) Femoral artery D) Greater saphenous vein

Ans: Greater saphenous vein Feedback: The greater saphenous vein is the most commonly used graft site for CABG. The right and left internal mammary arteries, radial arteries, and gastroepiploic artery are other graft sites used, though not as frequently. The femoral artery, brachial artery, and brachial vein are never harvested.

The nurse is caring for a patient who has undergone percutaneous transluminal coronary angioplasty (PTCA). What is the major indicator of success for this procedure? A) Increase in the size of the artery's lumen B) Decrease in arterial blood flow in relation to venous flow C) Increase in the patient's resting heart rate D) Increase in the patient's level of consciousness (LOC)

Ans: Increase in the size of the artery's lumen Feedback: PTCA is used to open blocked coronary vessels and resolve ischemia. The procedure may result in beneficial changes to the patient's LOC or heart rate, but these are not the overarching goals of PTCA. Increased arterial flow is the focus of the procedures.

The nurse is caring for an adult patient who had symptoms of unstable angina upon admission to the hospital. What nursing diagnosis underlies the discomfort associated with angina? A) Ineffective breathing pattern related to decreased cardiac output B) Anxiety related to fear of death C) Ineffective cardiopulmonary tissue perfusion related to coronary artery disease (CAD) D) Impaired skin integrity related to CAD

Ans: Ineffective cardiopulmonary tissue perfusion related to coronary artery disease (CAD) Feedback: Ineffective cardiopulmonary tissue perfusion directly results in the symptoms of discomfort associated with angina. Anxiety and ineffective breathing may result from angina chest pain, but they are not the causes. Skin integrity is not impaired by the effects of angina.

.The nurse is assessing a patient who was admitted to the critical care unit 3 hours ago following cardiac surgery. The nurse's most recent assessment reveals that the patient's left pedal pulses are not palpable and that the right pedal pulses are rated at +2. What is the nurse's best response? A) Document this expected assessment finding during the initial postoperative period. B) Reposition the patient with his left leg in a dependent position. C) Inform the patient's physician of this assessment finding. D) Administer an ordered dose of subcutaneous heparin.

Ans: Inform the patient's physician of this assessment finding. Feedback: If a pulse is absent in any extremity, the cause may be prior catheterization of that extremity, chronic peripheral vascular disease, or a thromboembolic obstruction. The nurse immediately reports newly identified absence of any pulse.

Preoperative education is an important part of the nursing care of patients having coronary artery revascularization. When explaining the pre- and postoperative regimens, the nurse would be sure to include education about which subject? A) Symptoms of hypovolemia B) Symptoms of low blood pressure C) Complications requiring graft removal D) Intubation and mechanical ventilation

Ans: Intubation and mechanical ventilation Feedback: Most patients remain intubated and on mechanical ventilation for several hours after surgery. It is important that patients realize that this will prevent them from talking, and the nurse should reassure them that the staff will be able to assist them with other means of communication. Teaching would generally not include symptoms of low blood pressure or hypovolemia, as these are not applicable to most patients. Teaching would also generally not include rare complications that would require graft removal.

The triage nurse in the ED assesses a 66-year-old male patient who presents to the ED with complaints of midsternal chest pain that has lasted for the last 5 hours. If the patient's symptoms are due to an MI, what will have happened to the myocardium? A) It may have developed an increased area of infarction during the time without treatment. B) It will probably not have more damage than if he came in immediately. C) It may be responsive to restoration of the area of dead cells with proper treatment. D) It has been irreparably damaged, so immediate treatment is no longer necessary.

Ans: It may have developed an increased area of infarction during the time without treatment. Feedback: When the patient experiences lack of oxygen to myocardium cells during an MI, the sooner treatment is initiated, the more likely the treatment will prevent or minimize myocardial tissue necrosis. Delays in treatment equate with increased myocardial damage. Despite the length of time the symptoms have been present, treatment needs to be initiated immediately to minimize further damage. Dead cells cannot be restored by any means.

The nurse is caring for a patient who has been diagnosed with an elevated cholesterol level. The nurse is aware that plaque on the inner lumen of arteries is composed chiefly of what? A) Lipids and fibrous tissue B) White blood cells C) Lipoproteins D) High-density cholesterol

Ans: Lipids and fibrous tissue Feedback: As T-lymphocytes and monocytes infiltrate to ingest lipids on the arterial wall and then die, a fibrous tissue develops. This causes plaques to form on the inner lumen of arterial walls. These plaques do not consist of white cells, lipoproteins, or high-density cholesterol.

The nurse is providing care for a patient with high cholesterol and triglyceride values. In teaching the patient about therapeutic lifestyle changes such as diet and exercise, the nurse realizes that the desired goal for cholesterol levels is which of the following? A) High HDL values and high triglyceride values B) Absence of detectable total cholesterol levels C) Elevated blood lipids, fasting glucose less than 100 D) Low LDL values and high HDL values

Ans: Low LDL values and high HDL values Feedback: The desired goal for cholesterol readings is for a patient to have low LDL and high HDL values. LDL exerts a harmful effect on the coronary vasculature because the small LDL particles can be easily transported into the vessel lining. In contrast, HDL promotes the use of total cholesterol by transporting LDL to the liver, where it is excreted. Elevated triglycerides are also a major risk factor for cardiovascular disease. A goal is also to keep triglyceride levels less than 150 mg/dL. All individuals possess detectable levels of total cholesterol.

An adult patient is admitted to the ED with chest pain. The patient states that he had developed unrelieved chest pain that was present for approximately 20 minutes before coming to the hospital. To minimize cardiac damage, the nurse should expect to administer which of the following interventions? A) Thrombolytics, oxygen administration, and nonsteroidal anti-inflammatories B) Morphine sulphate, oxygen, and bed rest C) Oxygen and beta-adrenergic blockers D) Bed rest, albuterol nebulizer treatments, and oxygen

Ans: Morphine sulphate, oxygen, and bed rest Feedback: The patient with suspected MI should immediately receive supplemental oxygen, aspirin, nitroglycerin, and morphine. Morphine sulphate reduces preload and decreases workload of the heart, along with increased oxygen from oxygen therapy and bed rest. With decreased cardiac demand, this provides the best chance of decreasing cardiac damage. NSAIDs and beta-blockers are not normally indicated. Albuterol, which is a medication used to manage asthma and respiratory conditions, will increase the heart rate.

An ED nurse is assessing an adult woman for a suspected MI. When planning the assessment, the nurse should be cognizant of what signs and symptoms of MI that are particularly common in female patients? Select all that apply. A) Shortness of breath B) Chest pain C) Anxiety D) Numbness E) Weakness

Ans: Numbness, Weakness Feedback: Although these symptoms are not wholly absent in men, many women have been found to have atypical symptoms of MI, including indigestion, nausea, palpitations, and numbness. Shortness of breath, chest pain, and anxiety are common symptoms of MI among patients of all ages and genders.

The nurse is providing an educational workshop about coronary artery disease (CAD) and its risk factors. The nurse explains to participants that CAD has many risk factors, some that can be controlled and some that cannot. What risk factors would the nurse list that can be controlled or modified? A) Gender, obesity, family history, and smoking B) Inactivity, stress, gender, and smoking C) Obesity, inactivity, diet, and smoking D) Stress, family history, and obesity

Ans: Obesity, inactivity, diet, and smoking Feedback: The risk factors for CAD that can be controlled or modified include obesity, inactivity, diet, stress, and smoking. Gender and family history are risk factors that cannot be controlled.

A patient is recovering in the hospital from cardiac surgery. The nurse has identified the diagnosis of risk for ineffective airway clearance related to pulmonary secretions. What intervention best addresses this risk? A) Administration of bronchodilators by nebulizer B) Administration of inhaled corticosteroids by metered dose inhaler (MDI) C) Patient's consistent performance of deep breathing and coughing exercises D) Patient's active participation in the cardiac rehabilitation program

Ans: Patient's consistent performance of deep breathing and coughing exercises Feedback: Clearance of pulmonary secretions is accomplished by frequent repositioning of the patient, suctioning, and chest physical therapy, as well as educating and encouraging the patient to breathe deeply and cough. Medications are not normally used to achieve this goal. Rehabilitation is important, but will not necessarily aid the mobilization of respiratory secretions.

The nurse is caring for patient who tells the nurse that he has an angina attack beginning. What is the nurse's most appropriate initial action? A) Have the patient sit down and put his head between his knees. B) Have the patient perform pursed-lip breathing. C) Have the patient stand still and bend over at the waist. D) Place the patient on bed rest in a semi-Fowler's position.

Ans: Place the patient on bed rest in a semi-Fowler's position. Feedback: When a patient experiences angina, the patient is directed to stop all activities and sit or rest in bed in a semi-Fowler's position to reduce the oxygen requirements of the ischemic myocardium. Pursed-lip breathing and standing will not reduce workload to the same extent. No need to have the patient put his head between his legs because cerebral perfusion is not lacking.

A patient presents to the ED in distress and complaining of "crushing" chest pain. What is the nurse's priority for assessment? A) Prompt initiation of an ECG B) Auscultation of the patient's point of maximal impulse (PMI) C) Rapid assessment of the patient's peripheral pulses D) Palpation of the patient's cardiac apex

Ans: Prompt initiation of an ECG Feedback: The 12-lead ECG provides information that assists in ruling out or diagnosing an acute MI. It should be obtained within 10 minutes from the time a patient reports pain or arrives in the ED. Each of the other listed assessments is valid, but ECG monitoring is the most time dependent priority.

A patient with cardiovascular disease is being treated with amlodipine (Norvasc), a calcium channel blocking agent. The therapeutic effects of calcium channel blockers include which of the following? A) Reducing the heart's workload by decreasing heart rate and myocardial contraction B) Preventing platelet aggregation and subsequent thrombosis C) Reducing myocardial oxygen consumption by blocking adrenergic stimulation to the heart D) Increasing the efficiency of myocardial oxygen consumption, thus decreasing ischemia and relieving pain

Ans: Reducing the heart's workload by decreasing heart rate and myocardial contraction Feedback: Calcium channel blocking agents decrease sinoatrial node automaticity and atrioventricular node conduction, resulting in a slower heart rate and a decrease in the strength of the heart muscle contraction. These effects decrease the workload of the heart. Antiplatelet and anticoagulation medications are administered to prevent platelet aggregation and subsequent thrombosis, which impedes blood flow. Beta-blockers reduce myocardial consumption by blocking beta-adrenergic sympathetic stimulation to the heart. The result is reduced myocardial contractility (force of contraction) to balance the myocardium oxygen needs and supply. Nitrates reduce myocardial oxygen consumption, which decreases ischemia and relieves pain by dilating the veins and, in higher doses, the arteries.

The nurse is caring for a patient who is believed to have just experienced an MI. The nurse notes changes in the ECG of the patient. What change on an ECG most strongly suggests to the nurse that ischemia is occurring? A) P wave inversion B) T wave inversion C) Q wave changes with no change in ST or T wave D) P wave enlargement

Ans: T wave inversion Feedback: T-wave inversion is an indicator of ischemic damage to myocardium. Typically, few changes to P waves occur during or after an MI, whereas Q-wave changes with no change in the ST or T wave indicate an old MI.

The nurse is assessing a woman who is pregnant at 27 weeks' gestation. The patient is concerned about the recent emergence of varicose veins on the backs of her calves. What is the nurse's best response? A) Facilitate a referral to a vascular surgeon. B) Assess the patient's ankle-brachial index (ABI) and perform Doppler ultrasound testing. C) Encourage the patient to increase her activity level. D) Teach the patient that circulatory changes during pregnancy frequently cause varicose veins.

Ans: Teach the patient that circulatory changes during pregnancy frequently cause varicose veins. Feedback: Pregnancy may cause varicosities because of hormonal effects related to decreased venous outflow, increased pressure by the gravid uterus, and increased blood volume. In most cases, no intervention or referral is necessary. This finding is not an indication for ABI assessment and increased activity will not likely resolve the problem.

The nurse is working with a patient who had an MI and is now active in rehabilitation. The nurse should teach this patient to cease activity if which of the following occurs? A) The patient experiences chest pain, palpitations, or dyspnea. B) The patient experiences a noticeable increase in heart rate during activity. C) The patient's oxygen saturation level drops below 96%. D) The patient's respiratory rate exceeds 30 breaths/min.

Ans: The patient experiences chest pain, palpitations, or dyspnea. Feedback: Any activity or exercise that causes dyspnea and chest pain should be stopped in the patient with CAD. Heart rate must not exceed the target rate, but an increase above resting rate is expected and is therapeutic. In most patients, a respiratory rate that exceeds 30 breaths/min is not problematic. Similarly, oxygen saturation slightly below 96% does not necessitate cessation of activity.

When assessing a patient diagnosed with angina pectoris it is most important for the nurse to gather what information? A) The patient's activities limitations and level of consciousness after the attacks B) The patient's symptoms and the activities that precipitate attacks C) The patient's understanding of the pathology of angina D) The patient's coping strategies surrounding the attacks

Ans: The patient's symptoms and the activities that precipitate attacks Feedback: The nurse must gather information about the patient's symptoms and activities, especially those that precede and precipitate attacks of angina pectoris. The patient's coping, understanding of the disease, and status following attacks are all important to know, but causative factors are a primary focus of the assessment interview.

Family members bring a patient to the ED with pale cool skin, sudden midsternal chest pain unrelieved with rest, and a history of CAD. How should the nurse best interpret these initial data? A) The symptoms indicate angina and should be treated as such. B) The symptoms indicate a pulmonary etiology rather than a cardiac etiology. C) The symptoms indicate an acute coronary episode and should be treated as such. D) Treatment should be determined pending the results of an exercise stress test.

Ans: The symptoms indicate an acute coronary episode and should be treated as such. Feedback: Angina and MI have similar symptoms and are considered the same process, but are on different points along a continuum. That the patient's symptoms are unrelieved by rest suggests an acute coronary episode rather than angina. Pale cool skin and sudden onset are inconsistent with a pulmonary etiology. Treatment should be initiated immediately regardless of diagnosis.

A patient with angina has been prescribed nitroglycerin. Before administering the drug, the nurse should inform the patient about what potential adverse effects? A) Nervousness or paresthesia B) Throbbing headache or dizziness C) Drowsiness or blurred vision D) Tinnitus or diplopia

Ans: Throbbing headache or dizziness Feedback: Headache and dizziness commonly occur when nitroglycerin is taken at the beginning of therapy. Nervousness, paresthesia, drowsiness, blurred vision, tinnitus, and diplopia do not typically occur as a result of nitroglycerin therapy.

The nurse is caring for a patient who is scheduled for cardiac surgery. What should the nurse include in preoperative care? A) With the patient, clarify the surgical procedure that will be performed. B) Withhold the patient's scheduled medications for at least 12 hours preoperatively. C) Inform the patient that health teaching will begin as soon as possible after surgery. D) Avoid discussing the patient's fears as not to exacerbate them.

Ans: With the patient, clarify the surgical procedure that will be performed. Feedback: Preoperatively, it is necessary to evaluate the patient's understanding of the surgical procedure, informed consent, and adherence to treatment protocols. Teaching would begin on admission or even prior to admission. The physician would write orders to alter the patient's medication regimen if necessary; this will vary from patient to patient. Fears should be addressed directly and empathically.

-recent intracranial pathology -current anticoagulation therapy -symptom onset greater than 3 hours prior to admission

As a member of the stroke team, the nurse knows that thrombolytic therapy carries the potential for benefit and for harm. The nurse should be cognizant of what contraindications for thrombolytic therapy? *select all that apply*

A nurse in a long-term care facility is caring for an 83-year-old woman who has a history of HF and peripheral arterial disease (PAD). At present the patient is unable to stand or ambulate. The nurse should implement measures to prevent what complication? A) Aoritis B) Deep vein thrombosis C) Thoracic aortic aneurysm D) Raynaud's disease

B

A patient converts from normal sinus rhythm at 80 bpm to atrial fibrillation with a ventricular response at 166 bpm. Blood pressure is 162/74 mm Hg. Respiratory rate is 20 breaths per minute with normal chest expansion and clear lungs bilaterally. IV heparin and Cardizem are given. The nurse caring for the patient understands that the main goal of treatment is what? A)Decrease SA node conduction B)Control ventricular heart rate C)Improve oxygenation D)Maintain anticoagulation

B

A patient has returned to the cardiac care unit after having a permanent pacemaker implantation. For which potential complication should the nurse most closely assess this patient? A)Chest pain B)Bleeding at the implantation site C)Malignant hyperthermia D)Bradycardia

B

A patient is scheduled for catheter ablation therapy. When describing this procedure to the patient's family, the nurse should address what aspect of the treatment? A)Resetting of the heart's contractility B)Destruction of specific cardiac cells C)Correction of structural cardiac abnormalities D)Clearance of partially occluded coronary arteries

B

A postsurgical patient has illuminated her call light to inform the nurse of a sudden onset of lower leg pain. On inspection, the nurse observes that the patient's left leg is visibly swollen and reddened. What is the nurse's most appropriate action? A) Administer a PRN dose of subcutaneous heparin. B) Inform the physician that the patient has signs and symptoms of VTE. C) Mobilize the patient promptly to dislodge any thrombi in the patient's lower leg. D) Massage the patient's lower leg to temporarily restore venous return.

B

An occupational health nurse is providing an educational event and has been asked by an administrative worker about the risk of varicose veins. What should the nurse suggest as a proactive preventative measure for varicose veins? A) Sit with crossed legs for a few minutes each hour to promote relaxation. B) Walk for several minutes every hour to promote circulation. C) Elevate the legs when tired. D) Wear snug-fitting ankle socks to decrease edema.

B

During a CPR class, a participant asks about the difference between cardioversion and defibrillation. What would be the instructor's best response? A)"Cardioversion is done on a beating heart; defibrillation is not." B)"The difference is the timing of the delivery of the electric current." C)"Defibrillation is synchronized with the electrical activity of the heart, but cardioversion is not." D)"Cardioversion is always attempted before defibrillation because it has fewer risks."

B

New nurses on the telemetry unit have been paired with preceptors. One new nurse asks her preceptor to explain depolarization. What would be the best answer by the preceptor? A)"Depolarization is the mechanical contraction of the heart muscles." B)"Depolarization is the electrical stimulation of the heart muscles." C)"Depolarization is the electrical relaxation of the heart muscles." D)"Depolarization is the mechanical relaxation of the heart muscles."

B

The ED nurse is caring for a patient who has gone into cardiac arrest. During external defibrillation, what action should the nurse perform? A)Place gel pads over the apex and posterior chest for better conduction. B)Ensure no one is touching the patient at the time shock is delivered. C)Continue to ventilate the patient via endotracheal tube during the procedure. D)Allow at least 3 minutes between shocks.

B

The nurse is caring for a 72-year-old patient who is in cardiac rehabilitation following heart surgery. The patient has been walking on a regular basis for about a week and walks for 15 minutes 3 times a day. The patient states that he is having a cramp-like pain in the legs every time he walks and that the pain gets "better when I rest." The patient's care plan should address what problem? A) Decreased mobility related to VTE B) Acute pain related to intermittent claudication C) Decreased mobility related to venous insufficiency D) Acute pain related to vasculitis

B

The nurse is caring for a patient who is admitted to the medical unit for the treatment of a venous ulcer in the area of her lateral malleolus that has been unresponsive to treatment. What is the nurse most likely to find during an assessment of this patient's wound? A) Hemorrhage B) Heavy exudate C) Deep wound bed D) Pale-colored wound bed

B

The nurse is caring for a patient with refractory atrial fibrillation who underwent the maze procedure several months ago. The nurse reviews the result of the patient's most recent cardiac imaging, which notes the presence of scarring on the atria. How should the nurse best respond to this finding? A)Recognize that the procedure was unsuccessful. B)Recognize this as a therapeutic goal of the procedure. CLiaise with the care team in preparation for repeating the maze procedure. D)Prepare the patient for pacemaker implantation.

B

The nurse is providing care for a patient who has just been diagnosed with peripheral arterial occlusive disease (PAD). What assessment finding is most consistent with this diagnosis? A) Numbness and tingling in the distal extremities B) Unequal peripheral pulses between extremities C) Visible clubbing of the fingers and toes D) Reddened extremities with muscle atrophy

B

The nurse is taking a health history of a new patient. The patient reports experiencing pain in his left lower leg and foot when walking. This pain is relieved with rest. The nurse notes that the left lower leg is slightly edematous and is hairless. When planning this patient's subsequent care, the nurse should most likely address what health problem? A) Coronary artery disease (CAD) B) Intermittent claudication C) Arterial embolus D) Raynaud's disease

B

The staff educator is teaching a CPR class. Which of the following aspects of defibrillation should the educator stress to the class? A)Apply the paddles directly to the patient's skin. B)Use a conducting medium between the paddles and the skin. C)Always use a petroleum-based gel between the paddles and the skin. D)Any available liquid can be used between the paddles and the skin.

B

When planning the care of a patient with an implanted pacemaker, what assessment should the nurse prioritize? A)Core body temperature B)Heart rate and rhythm C)Blood pressure D)Oxygen saturation level

B

While assessing a patient the nurse notes that the patient's ankle-brachial index (ABI) of the right leg is 0.40. How should the nurse best respond to this assessment finding? A) Assess the patient's use of over-the-counter dietary supplements. B) Implement interventions relevant to arterial narrowing. C) Encourage the patient to increase intake of foods high in vitamin K. D) Adjust the patient's activity level to accommodate decreased coronary output.

B

You are caring for a patient who is diagnosed with Raynaud's phenomenon. The nurse should plan interventions to address what nursing diagnosis? A) Chronic pain B) Ineffective tissue perfusion C) Impaired skin integrity D) Risk for injury

B

The prevention of VTE is an important part of the nursing care of high-risk patients. When providing patient teaching for these high-risk patients, the nurse should advise lifestyle changes, including which of the following? Select all that apply. A) High-protein diet B) Weight loss C) Regular exercise D) Smoking cessation E) Calcium and vitamin D supplementation

B,C,D

A 79-year-old man is admitted to the medical unit with digital gangrene. The man states that his problems first began when he stubbed his toe going to the bathroom in the dark. In addition to this trauma, the nurse should suspect that the patient has a history of what health problem? A) Raynaud's phenomenon B) CAD C) Arterial insufficiency D) Varicose veins

C

A medical nurse has admitted four patients over the course of a 12-hour shift. For which patient would assessment of ankle-brachial index (ABI) be most clearly warranted? A) A patient who has peripheral edema secondary to chronic heart failure B) An older adult patient who has a diagnosis of unstable angina C) A patient with poorly controlled type 1 diabetes who is a smoker D) A patient who has community-acquired pneumonia and a history of COPD

C

A nurse is admitting a 45-year-old man to the medical unit who has a history of PAD. While providing his health history, the patient reveals that he smokes about two packs of cigarettes a day, has a history of alcohol abuse, and does not exercise. What would be the priority health education for this patient? A) The lack of exercise, which is the main cause of PAD. B) The likelihood that heavy alcohol intake is a significant risk factor for PAD. C) Cigarettes contain nicotine, which is a powerful vasoconstrictor and may cause or aggravate PAD. D) Alcohol suppresses the immune system, creates high glucose levels, and may cause PAD.

C

A nurse is closely monitoring a patient who has recently been diagnosed with an abdominal aortic aneurysm. What assessment finding would signal an impending rupture of the patient's aneurysm? A) Sudden increase in blood pressure and a decrease in heart rate B) Cessation of pulsating in an aneurysm that has previously been pulsating visibly C) Sudden onset of severe back or abdominal pain D) New onset of hemoptysis

C

A patient has undergone diagnostic testing and received a diagnosis of sinus bradycardia attributable to sinus node dysfunction. When planning this patient's care, what nursing diagnosis is most appropriate? A)Acute pain B)Risk for unilateral neglect C)Risk for activity intolerance D)Risk for fluid volume excess

C

A patient is admitted to the cardiac care unit for an electrophysiology (EP) study. What goal should guide the planning and execution of the patient's care? A)Ablate the area causing the dysrhythmia. B)Freeze hypersensitive cells. C)Diagnose the dysrhythmia. D)Determine the nursing plan of care.

C

A patient presents to the clinic complaining of the inability to grasp objects with her right hand. The patient's right arm is cool and has a difference in blood pressure of more than 20 mm Hg compared with her left arm. The nurse should expect that the primary care provider may diagnose the woman with what health problem? A) Lymphedema B) Raynaud's phenomenon C) Upper extremity arterial occlusive disease D) Upper extremity VTE

C

An adult patient with third-degree AV block is admitted to the cardiac care unit and placed on continuous cardiac monitoring. What rhythm characteristic will the ECG most likely show? A)PP interval and RR interval are irregular. B)PP interval is equal to RR interval. C)Fewer QRS complexes than P waves D)PR interval is constant.

C

During a patient's care conference, the team is discussing whether the patient is a candidate for cardiac conduction surgery. What would be the most important criterion for a patient to have this surgery? A)Angina pectoris not responsive to other treatments B)Decreased activity tolerance related to decreased cardiac output C)Atrial and ventricular tachycardias not responsive to other treatments D)Ventricular fibrillation not responsive to other treatments

C

The nurse is caring for a patient who has had a dysrhythmic event. The nurse is aware of the need to assess for signs of diminished cardiac output (CO). What change in status may signal to the nurse a decrease in cardiac output? A)Increased blood pressure B)Bounding peripheral pulses C)Changes in level of consciousness D)Skin flushing

C

The nurse is caring for a patient who has just had an implantable cardioverter defibrillator (ICD) placed. What is the priority area for the nurse's assessment? A)Assessing the patient's activity level B)Facilitating transthoracic echocardiography C)Vigilant monitoring of the patient's ECG D)Close monitoring of the patient's peripheral perfusion

C

The nurse is caring for an acutely ill patient who is on anticoagulant therapy. The patient has a comorbidity of renal insufficiency. How will this patient's renal status affect heparin therapy? A) Heparin is contraindicated in the treatment of this patient. B) Heparin may be administered subcutaneously, but not IV. C) Lower doses of heparin are required for this patient. D) Coumadin will be substituted for heparin.

C

The nurse is writing a plan of care for a patient with a cardiac dysrhythmia. What would be the most appropriate goal for the patient? A)Maintain a resting heart rate below 70 bpm. B)Maintain adequate control of chest pain. C)Maintain adequate cardiac output. D)Maintain normal cardiac structure.

C

The nursing educator is presenting a case study of an adult patient who has abnormal ventricular depolarization. This pathologic change would be most evident in what component of the ECG? A)P wave B)T wave C)QRS complex D)U wave

C

A group of nurses are participating in orientation to a telemetry unit. What should the staff educator tell this class about ST segments? A)They are the part of an ECG that reflects systole. B)They are the part of an ECG used to calculate ventricular rate and rhythm. C)They are the part of an ECG that reflects the time from ventricular depolarization through repolarization. D)They are the part of an ECG that represents early ventricular repolarization.

D

A nurse in the rehabilitation unit is caring for an older adult patient who is in cardiac rehabilitation following an MI. The nurse's plan of care calls for the patient to walk for 10 minutes 3 times a day. The patient questions the relationship between walking and heart function. How should the nurse best reply? A) "The arteries in your legs constrict when you walk and allow the blood to move faster and with more pressure on the tissue." B) Walking increases your heart rate and blood pressure. Therefore your heart is under less stress." C) "Walking helps your heart adjust to your new arteries and helps build your self-esteem." D) "When you walk, the muscles in your legs contract and pump the blood in your veins back toward your heart, which allows more blood to return to your heart."

D

A nurse is assessing a new patient who is diagnosed with PAD. The nurse cannot feel the pulse in the patient's left foot. How should the nurse proceed with assessment? A) Have the primary care provider order a CT. B) Apply a tourniquet for 3 to 5 minutes and then reassess. C) Elevate the extremity and attempt to palpate the pulses. D) Use Doppler ultrasound to identify the pulses.

D

A nurse is providing health education to a patient scheduled for cryoablation therapy. The nurse should describe what aspect of this treatment? A)Peeling away the area of endocardium responsible for the dysrhythmia B)Using electrical shocks directly to the endocarduim to eliminate the source of dysrhythmia C)Using high-frequency sound waves to eliminate the source of dysrhythmia D)Using a cooled probe to eliminate the source of dysrhythmia

D

A nurse is reviewing the physiological factors that affect a patient's cardiovascular health and tissue oxygenation. What is the systemic arteriovenous oxygen difference? A) The average amount of oxygen removed by each organ in the body B) The amount of oxygen removed from the blood by the heart C) The amount of oxygen returning to the lungs via the pulmonary artery D) The amount of oxygen in aortic blood minus the amount of oxygen in the vena caval blood

D

A nurse working in a long-term care facility is performing the admission assessment of a newly admitted, 85-year-old resident. During inspection of the resident's feet, the nurse notes that she appears to have early evidence of gangrene on one of her great toes. The nurse knows that gangrene in the elderly is often the first sign of what? A) Chronic venous insufficiency B) Raynaud's phenomenon C) VTE D) PAD

D

A patient calls his cardiologist's office and talks to the nurse. He is concerned because he feels he is being defibrillated too often. The nurse tells the patient to come to the office to be evaluated because the nurse knows that the most frequent complication of ICD therapy is what? A)Infection B)Failure to capture C)Premature battery depletion D)Oversensing of dysrhythmias

D

A patient comes to the walk-in clinic with complaints of pain in his foot following stepping on a roofing nail 4 days ago. The patient has a visible red streak running up his foot and ankle. What health problem should the nurse suspect? A) Cellulitis B) Local inflammation C) Elephantiasis D) Lymphangitis

D

An ECG has been ordered for a newly admitted patient. What should the nurse do prior to electrode placement? A)Clean the skin with providone-iodine solution. B)Ensure that the area for electrode placement is dry. C)Apply tincture of benzoin to the electrode sites and wait for it to become "tacky." D)Gently abrade the skin by rubbing the electrode sites with dry gauze or cloth.

D

An older adult patient has been treated for a venous ulcer and a plan is in place to prevent the occurrence of future ulcers. What should the nurse include in this plan? A) Use of supplementary oxygen to aid tissue oxygenation B) Daily use of normal saline compresses on the lower limbs C) Daily administration of prophylactic antibiotics D) A high-protein diet that is rich in vitamins

D

Following cardiac resuscitation, a patient has been placed in a state of mild hypothermia before being transferred to the cardiac intensive care unit. The nurse's assessment reveals that the patient is experiencing neuromuscular paralysis. How should the nurse best respond? A)Administer hypertonic IV solution. B)Administer a bolus of warned normal saline. C)Reassess the patient in 15 minutes. D)Document this as an expected assessment finding.

D

Graduated compression stockings have been prescribed to treat a patient's venous insufficiency. What education should the nurse prioritize when introducing this intervention to the patient? A) The need to take anticoagulants concurrent with using compression stockings B) The need to wear the stockings on a "one day on, one day off" schedule C) The importance of wearing the stockings around the clock to ensure maximum benefit D) The importance of ensuring the stockings are applied evenly with no pressure points

D

How should the nurse best position a patient who has leg ulcers that are venous in origin? A) Keep the patient's legs flat and straight. B) Keep the patient's knees bent to 45-degree angle and supported with pillows. C) Elevate the patient's lower extremities. D) Dangle the patient's legs over the side of the bed.

D

The nurse caring for a patient whose sudden onset of sinus bradycardia is not responding adequately to atropine. What might be the treatment of choice for this patient? A)Implanted pacemaker B)Trancutaneous pacemaker C)ICD D)Asynchronous defibrillator

D

The nurse caring for a patient with a leg ulcer has finished assessing the patient and is developing a problem list prior to writing a plan of care. What major nursing diagnosis might the care plan include? A) Risk for disuse syndrome B) Ineffective health maintenance C) Sedentary lifestyle D) Imbalanced nutrition: less than body requirements

D

The nurse is assessing a patient who had a pacemaker implanted 4 weeks ago. During the patient's most recent follow-up appointment, the nurse identifies data that suggest the patient may be socially isolated and depressed. What nursing diagnosis is suggested by these data? A)Decisional conflict related to pacemaker implantation B)Deficient knowledge related to pacemaker implantation C)Spiritual distress related to pacemaker implantation D)Ineffective coping related to pacemaker implantation

D

The nurse is caring for a patient on telemetry. The patient's ECG shows a shortened PR interval, slurring of the initial QRS deflection, and prolonged QRS duration. What does this ECG show? A)Sinus bradycardia B)Myocardial infarction C)Lupus-like syndrome D)Wolf-Parkinson-White (WPW) syndrome

D

The nurse is providing care to a patient who has just undergone an electrophysiologic (EP) study. The patient states that she is nervous about "things going wrong" during the procedure. What is the nurse's best response? A)"This is basically a risk-free procedure." B)"Thousands of patients undergo EP every year." C)"Remember that this is a step that will bring you closer to enjoying good health." D)"The whole team will be monitoring you very closely for the entire procedure."

D

alteration in LOC

The nurse is discharging home a patient who had a hemorrhagic stroke, close monitoring of vital signs & neurologic changes is imperative. What is the earliest sign of deterioration in a patient with a hemorrhagic stroke of which the nurse should be aware?

provide a board of commonly used needs and phrases

The pt has been dx with aphasia after suffering a stroke. What can the nurse do to best make the patients atmosphere more conducive to communication?

have a colleague follow the patient closely with a wheelchair

a patient has recently begun mobilizing during the recovery from an ischemic stroke. To protect the patients safety during mobilization, the nurse should perform what action?

the patient should mobilize as soon as she is physically able

a patient who has experienced an ischemic stroke has been admitted to the medical unit. The patients family is adamant that she remain on bed rest to haste her recovery and to conserve energy. What principle of care should inform the nurses response to the family.

the patient should be approached on the side where visual perception is intact

a patient who suffered an ischemic stroke now has disturbed sensory perception. What principle should guide the nurses care of this patient?

bleeding

a patient with a new diagnosis of ischemic stroke is deemed to be a candidate for treatment with tissue plasminogen activator (t-PA) and has been admitted to the ICU. In addition to closely monitoring the patients cardiac & neuro status, the nurse monitors the patient for signs of what complication?

the focus on care in a rehabilitation facility is to help the patient to resume as much self-care as possible

a rehabilitation nurse caring for a patient who has had a stroke is approached by the patients family and asked why the patient has to do so much for herself when she is obviously struggling. What would be the nurses best answer?

positioning to avoid hypoxia

after a major ischemic stroke, a possible complication is cerebral edema. Nursing care during the immediate recovery period from an ischemic stroke should include which of the following?

supervise the patients activities of daily living closely

following dx testing, a patient has been admitted to the ICU and placed on cerebral aneurysm precautions. what nursing action should be included in patients plan of care?

white male, age 60, with hx of uncontrolled HTN

the nurse is performing stroke risk screenings at a hospital open house. The nurse has identified four patients who might be at risk for stroke. Which patient is likely at the highest risk for hemorrhagic stroke?

take antihypertensive meds as ordered

the nurse is preparing health education for a patient who is being discharged after hospitalization for a hemorrhagic stroke. What content should the nurse include in this education?

Aspirin 81 mg PO

the nurse is reviewing the medication administration record of a female patient who possesses numerous risk factors for stroke. Which of the womans medications carries the greatest potential for reducing her risk of stroke?

13%

the public health nurse is planning a health promotion campaign that reflects current epidemiologic trends. The nurse should know that hemorrhagic stroke currently accounts for what percentage of total strokes in the US?

frustration around changes in function and communication

when preparing to discharge a patient home, the nurse has met with the family and warned them that the patient may exhibit unexpected emotional responses. The nurse should teach the family that these responses are typically a result of what cause?


संबंधित स्टडी सेट्स

Commercial Property Insurance, Part 2

View Set

The Electromagnetic Spectrum (EMS): Waves of energy

View Set

Policing in America: Chapters 11-13

View Set

AHT 3 - Shock - ATI, Lewis, Evolve. MOD 1- Exam 1

View Set

Lab 3: Goldmann Application Tonometry

View Set

Middle Eastern History and Culture Test #1

View Set